You are on page 1of 75

Total Marks : 200

Online Prelims TEST - 26 (TEXTBOOK)


( InsightsIAS Mock Test Series for UPSC Preliminary Exam 2020 ) Mark Scored : 0

1 Agrobacterium has become an important tool for plant improvement by genetic engineering. This is
because
1. It is a Gram-positive bacterium which does not has cell walls.
2. It is well known for its ability to transfer DNA between itself and plants.

Select the correct answer using the codes below.


A. 1 only
B. 2 only
C. Both 1 and 2
D. None of the above

Correct Answer : B

Answer Justification :

Justification: Agrobacterium tumafciens is a Gram-negative bacterium that uses horizontal gene


transfer to cause tumors in plants.

Agrobacterium is well known for its ability to transfer DNA between itself and plants, and for
this reason it has become an important tool for plant improvement by genetic engineering.

When the bacterial DNA is integrated into a plant chromosome, it becomes part of the plants
chromosomes and the integrated DNA sequence stably expressed and inherited.

These bacteria have both resident DNA (that remains within the bacteria) and plasmid DNA
(that is able to travel outside the bacteria into other cells).

This natural phenomenon is exploited by scientists to introduce the genes of interest into
plant genome.

Sweet potatoes from all over the world naturally contain genes from the bacterium
Agrobacterium.

Q Source: Based on past year UPSC papers

2 The Pacific decadal oscillation (PDO) differs from the El Niño–Southern Oscillation (ENSO) in that
1. PDO does not affect Monsoon activity in India unlike ENSO.
2. PDO is associated only with cold cycles (cold water events) unlike ENSO which has both warm and
cold cycles.

Select the correct answer using the codes below.


A. 1 only

1
Total Marks : 200
Online Prelims TEST - 26 (TEXTBOOK)
( InsightsIAS Mock Test Series for UPSC Preliminary Exam 2020 ) Mark Scored : 0

B. 2 only
C. Both 1 and 2
D. None of the above

Correct Answer : D

Answer Justification :

Justification: Statement 1: Northeast India, one of the wettest places on the Earth has been
experiencing rapid drying, especially in the last 30 years. The decreasing monsoon rainfall is
associated with natural changes in the Pacific decadal oscillation (PDO).

The “Pacific Decadal Oscillation” (PDO) is a long-lived El Niño-like pattern of Pacific climate
variability. The PDO is detected as warm or cool surface waters in the Pacific Ocean, north of 20°N.
Major changes in northeast Pacific marine ecosystems have been correlated with phase changes in
the PDO; warm eras have seen enhanced coastal ocean biological productivity in Alaska and
inhibited productivity off the west coast of the contiguous United States, while cold PDO areas have
seen the opposite north-south pattern of marine ecosystem productivity.

Statement 2: How is it different from ENSO?

Duration: Both PDO and ENSO have similar spatial climate fingerprints yet the major difference is
that PDO persists for 20-30 years while the typical ENSO persists for 6 to 18 months. The primary
climatic fingerprints of the PDO are most visible in the North Pacific/North American sector, while
secondary signatures exist in the tropics. On the contrary, the primary climatic fingerprints of the
ENSO are visible in tropics while secondary are visible in North Pacific/North American sector.

The PDO has two cycles, viz. Cold Cycle and Warm Cycle, very much similar to La Nina and
El Nino of the ENSO cycle. Influences: The PDO has a major influence on Alaskan and for those
matter global temperatures.

The positive phase favors more El Ninos and a stronger Aleutian low and warm water in the north
Pacific off the Alaskan coast. The negative phase more La Ninas and cold eastern Gulf of Alaska
waters. Note the strong similarity of the positive phase with El Nino and the negative with La Nina.
PDO is responsible for bringing colder surface water temperatures and thus beginning the overall
cooling effect in recent times in Alaska.

Q Source:
https://www.insightsonindia.com/wp-content/uploads/2019/06/InsightsonIndia-May-2019-Current-Af
fairs-final.pdf

3 The Chairman of the Public Accounts Committee (PAC) of the Parliament is appointed by the speaker
from amongst

A. Members of both Lok Sabha and Rajya Sabha


B. Members of Lok Sabha only
C. Members of Rajya Sabha only

2
Total Marks : 200
Online Prelims TEST - 26 (TEXTBOOK)
( InsightsIAS Mock Test Series for UPSC Preliminary Exam 2020 ) Mark Scored : 0

D. Lok Sabha Secretariat

Correct Answer : A

Answer Justification :

Learning: The chairman of the committee is appointed by the Speaker from amongst its members
(both LS and RS).

The members are elected by the Parliament every year from amongst its members according
to the principle of proportional representation by means of the single transferable vote.

Until 1966–67, the chairman of the committee belonged to the ruling party. However, since
1967 a convention has developed whereby the chairman of the committee is selected
invariably from the Opposition.

The function of the committee is to examine the annual audit reports of the CAG.

Q Source: Chapter on Parliament: Indian Polity: M Laxmikanth

4 The protected area that is situated at the tri-junction of states of Tamil Nadu, Kerala and Karnataka is

A. Eravikulam National Park


B. Silent Valley National Park
C. Mudumalai Wildlife Sanctuary
D. Periyar Wildlife Sanctuary

Correct Answer : C

Answer Justification :

Learning: Mudumalai Wildlife Sanctuary & National Park is situated at this tri-junction on the
North Eastern Slopes of the Nilgiris part of Western Ghats descending to the Mysore Plateau.

With Bandipur Tiger Reserve (Karnataka) in the north, and Wynad Wildlife Sanctuary (Kerala) in the
west the region forms a single, continuous viable habitat for a varied range of wildlife and is a part
of the Nilgiri Biosphere Reserve.

While Mudumalai’s western half receives the southwest monsoon, the eastern tracts receive the
relatively gentler north-east monsoon which results in a diversity of vegetation types and typical
migration of herbivores.

Q Source: National parks of India

3
Total Marks : 200
Online Prelims TEST - 26 (TEXTBOOK)
( InsightsIAS Mock Test Series for UPSC Preliminary Exam 2020 ) Mark Scored : 0

5 Consider the following about currency chests.


1. The responsibility for managing the currency in circulation is vested in the RBI.
2. The central bank advises the Centre on the number of notes to be printed, the currency
denominations and their security features.
3. Currency chests are branches of selected banks authorised by the RBI to stock rupee notes and
coins.

Select the correct answer using the codes below.


A. 1 only
B. 2 only
C. 1, 2 and 3
D. 1 and 2 only

Correct Answer : C

Answer Justification :

Justification: Statement 1 and 2: The responsibility for managing the currency in circulation is
vested in the RBI. The central bank advises the Centre on the number of notes to be printed, the
currency denominations, security features and so on. The number of notes that need to be printed is
determined using a statistical model that takes the pace of economic growth, rate of inflation and
the replacement rate of soiled notes. The Government has, however, reserved the right to determine
the amount of coins that have to be minted.

Statement 3: Currency chests are branches of selected banks authorised by the RBI to stock rupee
notes and coins.

The Reserve Bank of India is planning to allow large modern currency chests to increase the service
charges on cash deposited by non-chest bank branches from the existing rate of ₹5 per packet of
100 pieces to a higher rate subject to a maximum of ₹8 per packet. For this, only a currency chest
(CC) that fulfils the minimum standards will be eligible to be classified as a large modern CC.

Q Source:
https://www.insightsonindia.com/wp-content/uploads/2019/06/InsightsonIndia-May-2019-Current-Af
fairs-final.pdf

6 He served as the judge of Bombay High Court in British India; inaugurated the Indian National Social
Conference, which for many years met for its annual sessions alongside the Indian National Congress;
and was also a member of Bombay Legislative Council. He is

A. Keshub Chandra Sen


B. Baba Amte
C. Mahadev Govind Ranade
D. Arbondo Ghosh

Correct Answer : C

4
Total Marks : 200
Online Prelims TEST - 26 (TEXTBOOK)
( InsightsIAS Mock Test Series for UPSC Preliminary Exam 2020 ) Mark Scored : 0

Answer Justification :

Learning: Ranade’s principal forte was social and religious reform. He worked unceasingly for the
eradication of child marriage, the purdah system and other such ills.

During his life he helped to establish the Vaktruttvottejak Sabha, the Poona Sarvajanik Sabha and
the Prarthana Samaj, and would edit a Bombay Anglo-Marathi daily paper, the Induprakash,
founded on his ideology of social and religious reform.

He served as member of the Bombay legislative council, member of the finance committee at the
centre, and the judge of Bombay High Court.

Q Source: 12th Tamil Nadu History Textbook

7 Consider the following matches of Himalayan ranges and


their peaks/ranges:
1. Himadri: A. Makalu
2. Himachal: B. Dhaula Dhar
3. Shiwalik: C. Dhang range

Select the correct answer using the codes below.


A. 1C, 2A, 3B
B. 1A, 2B, 3C
C. 1 & 2 – A, 3C
D. 1B, 2 & 3 - A

Correct Answer : B

Answer Justification :

Justification: Statement 1: Known as Greater Himalayas, its average elevation is six thousand
metre.

It is the most continuous range, snow bound and many glaciers descend from this range. It has high
peaks like Mt. Everest, Kanchenjunga, Makalu, Dhaulagiri, Nanga Parbat etc. having a height of
more than 8000 metres.

Statement 2: The altitude of this range lies between 1000 and 4500 metres and the average width is
50 KM.

The Prominent ranges in this are Pir Panjal, Dhaula Dhar and Mahabharata ranges. It compresses of
many famous hill stations like Shimla, Dalhousie Darjeeling, Chakrata, Mussoorie, Nanital etc.

Statement 3: It is the outer most range of the Himalayas. The altitude varies between 900-1100
meters and the width ranges between 10-50 KM. They have low hills like Jammu Hills, etc. The
valleys lying between Siwalik and Lesser Himalayas (Himachal) are called ‘Duns’ like Dehra Dun,

5
Total Marks : 200
Online Prelims TEST - 26 (TEXTBOOK)
( InsightsIAS Mock Test Series for UPSC Preliminary Exam 2020 ) Mark Scored : 0

Kotli Dun and Patli Dun.

Q Source: 9th NCERT: Geography

8 Presently, a ‘Kilogram’ is defined by

A. Weight of a preserved platinum rod


B. Amount of electricity needed by an electromagnet to counteract gravitational force
C. Movement of quartz in heavy quantum fields
D. Gold standard as maintained by the crown

Correct Answer : B

Answer Justification :

Justification: Scientists, last year, have changed the way the kilogram is defined. The decision was
made at the General Conference on Weights and Measures. The new definitions came into force on
20 May 2019. Why kill off the kilogram? Currently, it is defined by the weight of a platinum-based
ingot called “Le Grand K” which is locked away in a safe in Paris.

In a world where accurate measurement is now critical in many areas, such as in drug development,
nanotechnology and precision engineering – those responsible for maintaining the international
system had no option but to move beyond Le Grand K to a more robust definition.

How wrong is Le Grand K?

The fluctuation is about 50 parts in a billion, less than the weight of a single eyelash. But although it
is tiny, the change can have important consequences.

How does the new system work?

Electromagnets generate a force. Scrap-yards use them on cranes to lift and move large metal
objects, such as old cars. The pull of the electromagnet, the force it exerts, is directly related to the
amount of electrical current going through its coils. There is, therefore, a direct relationship
between electricity and weight. So, in principle, scientists can define a kilogram, or any other
weight, in terms of the amount of electricity needed to counteract the weight (gravitational force
acting on a mass).

The CSIR-NPL, which is India’s official reference keeper of units of measurements has released a
set of recommendations requiring that school textbooks, engineering-education books, and course
curriculum update the definition of the kilogram. The institute is also in the process of making its
own ‘Kibble Balance’, a device that was used to measure the Planck Constant and thereby reboot
the kilogram.

Q Source: In news

9 In legislative matters, Rajya Sabha enjoys an equal Status with Lok Sabha in

6
Total Marks : 200
Online Prelims TEST - 26 (TEXTBOOK)
( InsightsIAS Mock Test Series for UPSC Preliminary Exam 2020 ) Mark Scored : 0

1. Making recommendation to the President for the removal of Chief Justice and judges of Supreme
Court and high courts
2. Introduction and passage of certain financial bills involving expenditure from the Consolidated Fund
of India
3. Approval of proclamation of all three types of emergencies by the President
4. Election and removal of the Vice-President

Select the correct answer using the codes below.


A. 1 and 4 only
B. 2, 3 and 4 only
C. 2 and 3 only
D. 1, 2, 3 and 4

Correct Answer : D

Answer Justification :

Justification: Some matters where the powers and status of the Rajya Sabha are equal to that of
the Lok Sabha:

Introduction and passage of ordinary bills, constitutional amendment bills; and financial bills
involving expenditure from the Consolidated Fund of India.

Election and impeachment of the president.

Election and removal of the Vice-President. However, Rajya Sabha alone can initiate the
removal of the vice-president.

Making recommendation to the President for the removal of Chief Justice and judges of
Supreme Court and high courts, chief election commissioner and comptroller and auditor
general.

Approval of proclamation of all three types of emergencies by the President.

Consideration of reports of constitutional bodies like Finance Commission

Enlargement of the jurisdiction of SC and the UPSC

Q Source: Chapter on Parliament: Indian Polity: M Laxmikanth

10 RISAT series satellites are used by ISRO for

7
Total Marks : 200
Online Prelims TEST - 26 (TEXTBOOK)
( InsightsIAS Mock Test Series for UPSC Preliminary Exam 2020 ) Mark Scored : 0

1. Reconnaissance
2. Strategic surveillance
3. Disaster management

Select the correct answer using the codes below.


A. 3 only
B. 2 only
C. 1 and 2 only
D. 1, 2 and 3

Correct Answer : D

Answer Justification :

Justification: ISRO successfully launched radar imaging satellite RISAT-2B on-board PSLV-C46
from Sriharikota. It has been developed for military and general surveillance purposes. The data
will also be used in fields of agriculture, forestry and disaster management support. RISAT-2B is
equipped with a synthetic aperture radar that can take pictures of the earth during day and night,
and also under cloudy conditions.

RISAT-2B was placed into an orbit of 555 km with an inclination of 37 degree to the equator. This is
the fourth flight unit of the RISAT programme and it would be used for reconnaissance, strategic
surveillance and disaster management. Reconnaissance is military observation of a region to locate
an enemy or ascertain strategic features.

With this advanced earth observation satellite, ISRO has introduced a complex new technology.
That is a 3.6 metre unfurlable radial rib antenna. This is also going to be the technology of the
future. RISAT-2B is going to RISAT-2, which was placed in the orbit in 2009. RISAT-2 was RISAT-1’s
replacement, a microwave remote sensing satellite that was launched in 2012. RISAT-2 was actively
used India to monitor activities in camps across the border in Pakistan to thwart infiltration bids by
terrorists.

Q Source:
https://www.insightsonindia.com/wp-content/uploads/2019/06/InsightsonIndia-May-2019-Current-Af
fairs-final.pdf

11 Consider the following statements.


1. Daskathia is a form of ballad singing prevalent in Odisha.
2. The performers generally narrate a poem of mythological or religious intent in Daskathia.

Which of the above is/are correct?


A. 1 only
B. 2 only
C. Both 1 and 2
D. None

8
Total Marks : 200
Online Prelims TEST - 26 (TEXTBOOK)
( InsightsIAS Mock Test Series for UPSC Preliminary Exam 2020 ) Mark Scored : 0

Correct Answer : C

Answer Justification :

Learning: It is an art form that’s exclusive to Orissa, Daskathia is performed usually by two men,
the singer and his assistant and is simpler than a Pala.

Daskathia is a name derived from a unique musical instrument called “Kathi” or “Ram Tali”,
wooden clappers used during the presentation.

The performance is a form of worship and offering on behalf of the “Das”, the devotee.

The performers - the Gayak (singer) and his accomplice (Palia) generally narrate a poem of
mythological or religious intent.

Q Source: CCRT: Performing Arts

12 Consider the following about White tigers.


1. They are a genetic hybrid of Bengal Tiger and Siberian Tiger.
2. They are extinct in the wild in India.

Select the correct answer using the codes below.


A. 1 only
B. 2 only
C. Both 1 and 2
D. None of the above

Correct Answer : B

Answer Justification :

Justification: The last captive white tiger named Bajirao died recently at Sanjay Gandhi National
Park, Mumbai. The tiger was born at National Park in 2001.

White tigers are not a separate sub species of Tiger. The White colour is basically due to a
pigmentation The White colour is due to lack of red or yellow pheomelanin pigment and the
presence of unique recessive genes.

White tigers were once found in Madhya Pradesh, Assam, West Bengal, Bihar. Now there are no
white tigers in the wild.

The last white tiger reported in wild was captured in Rewa forest Madhya Pradesh. Sanjay Gandhi
National Park is in Maharashtra. It is located in Mumbai. It has a protected archaeological site
called Kanheri caves.

9
Total Marks : 200
Online Prelims TEST - 26 (TEXTBOOK)
( InsightsIAS Mock Test Series for UPSC Preliminary Exam 2020 ) Mark Scored : 0

Q Source: In news:
https://timesofindia.indiatimes.com/city/bhopal/white-tigers-in-wild-wii-says-yes-after-no-activists-se
e-red/articleshow/72627533.cms

13 Consider the following statements.


1. During the northeast monsoon, strong upwelling occurs along the western coast of India.
2. The nutrient-rich upwelled water stimulates the growth and reproduction of primary producers such
as phytoplankton.

Select the correct answer using the codes below.


A. 1 only
B. 2 only
C. Both 1 and 2
D. None of the above

Correct Answer : C

Answer Justification :

Justification: Statement 1: Upwelling is a seasonal phenomenon associated with the monsoon. It


implies churning of the ocean water due to movement of the wind.

It involves wind-driven motion of dense, cooler, and usually nutrient-rich water towards the ocean
surface, replacing the warmer, usually nutrient-depleted surface water. The nutrient-rich upwelled
water stimulates the growth and reproduction of primary producers such as phytoplankton.
Therefore, it sees the migration of deep sea oxygen minimum zone (OMZ) towards the coast.

During the northeast monsoon, strong upwelling occurs along the western coast of India.

During the southwest monsoon, upwelling occurs off the Somali and Arabian coasts and south of
Java. It is most intense between 5° and 11° N, with replacement of warmer surface water by water
of about 14 °C.

Q Source: Based on past year UPSC papers

14 Consider the following statements.


1. The aggrieved party can move either Supreme Court or High Court in case of fundamental right
violations, but only Supreme Court in case of federal disputes.
2. The Supreme Court can issue writs for the settlement of federal disputes, but not those related to
fundamental rights.

Which of the above is/are correct?


A. 1 only
B. 2 only
C. Both 1 and 2
D. None

10
Total Marks : 200
Online Prelims TEST - 26 (TEXTBOOK)
( InsightsIAS Mock Test Series for UPSC Preliminary Exam 2020 ) Mark Scored : 0

Correct Answer : A

Answer Justification :

Justification: The original jurisdiction of the Supreme Court with regard to federal disputes is
different from its original jurisdiction with regard to disputes relating to fundamental rights.

Statement 1: The federal dispute jurisdiction is exclusive to the Supreme Court. It cannot be raised
in the High Courts.

But, when the Fundamental Rights of a citizen are violated, the aggrieved party has the option of
moving either the high court or the Supreme Court directly.

In this case, it is concurrent with high courts jurisdiction.

Statement 2: There is also a difference between the writ jurisdiction of the Supreme Court and that
of the high court.

The Supreme Court can issue writs only for the enforcement of the Fundamental Rights and not for
other purposes. The high court, on the other hand, can issue writs not only for the enforcement of
the fundamental rights but also for other purposes.

Q Source: Chapter on Supreme Court: Indian Polity: M Laxmikanth

15 The ‘Farmers’ parliament’, the first of its kind in the country, was organised sometime back in the
state of

A. Madhya Pradesh
B. Kerala
C. Assam
D. Meghalaya

Correct Answer : D

Answer Justification :

Justification: The Meghalaya government has approved setting up of a commission to look into the
problems faced by farmers. The proposal was made by the Agriculture Department in line with the
resolution passed at the ‘farmers’ parliament’. The ‘farmers’ parliament’, the first of its kind in the
country, was organised in Meghalaya and attended by scientists, bureaucrats, apart from farmers,
to discuss issues related to the agriculture situation in the State.

Learning: Just as the national ‘Kisan Mukti’ protest rally in New Delhi concluded, the Meghalaya
government held its first-ever farmers' parliament between December 4 and 5, 2018, at the state
convention centre in capital Shillong.

11
Total Marks : 200
Online Prelims TEST - 26 (TEXTBOOK)
( InsightsIAS Mock Test Series for UPSC Preliminary Exam 2020 ) Mark Scored : 0

The parliament was organised by the state department of agriculture in collaboration with
Meghalaya Basin Development Authority (MBDA) and the Hill Farmers' Union (HFU).

Addressing the gathering of over 500 farmer representatives from across the state, Chief
Minister Conrad Sangma acknowledged the failure of his government to listen to its farmers’
problems.

“Thus far, the policies and schemes had been formulated without knowing what the farmers
truly needed. And the farmers continued to be unaware of the government's schemes and
policies,” he said.

Agriculture production commissioner of Meghalaya, IAS officer KN Kumar, corroborated this


lack of communication between the government and the farmers, saying, “This [information]
space was being filled by middlemen and weak farmers associations,

Each of the 500 farmer representatives at the parliament represented their respective block's
marginal, small, and semi-medium scale farmers that account for 96 per cent of the farmers in
Meghalaya.

The focus of the two-day gathering was mutually constructive dialogue without party politics.
The advisor and chief strategist, Lanu Ignatius, explained, “The meeting had well structured
and moderated sessions, with ample time for questions and answers.”

The Meghalaya farmers demanded that the state government and MPs support the two
national Kisan Mukti Bills when they are tabled. These bills seek to comprehensively end the
vicious cycle of farmer's debt by synergising the solutions of farmers debt relief with solutions
to provide them remunerative minimum support prices as per the reports by the MS
Swaminathan commission-led National Commission on Farmers.

All the demands were consolidated as a charter of demands and resolution at the farmer
parliament. This charter has been submitted to the state Cabinet—to be discussed in a special
state legislative assembly session.

Q Source:
https://www.downtoearth.org.in/blog/agriculture/meghalaya-farmer-parliament-will-it-help-plug-poli
cy-loopholes-62938

16 In the context of Modern India, Jugantar and Bharathmatha Association were

A. Revivalist Hindu organizations


B. Publications against colonialism
C. Social non-profit organizations

12
Total Marks : 200
Online Prelims TEST - 26 (TEXTBOOK)
( InsightsIAS Mock Test Series for UPSC Preliminary Exam 2020 ) Mark Scored : 0

D. Revolutionary groups

Correct Answer : D

Answer Justification :

Learning: In the first half of the 20th century, revolutionary groups sprang up mainly in Bengal,
Maharashtra, Punjab and Madras.

The revolutionaries were not satisfied with the methods of both the moderates and extremists.
Hence, they started many revolutionary secret organizations.

In Bengal Anusilan Samiti and Jugantar were established. In Maharashtra Savarkar brothers
had set up Abhinava Bharat. In the Madras Presidency, Bharathmatha Association was started
by Nilakanta Bramachari.

In Punjab Ajit Singh set up a secret society to spread revolutionary ideas among the youth –
Ghadr party.

Q Source: 12th Standard TamilNadu History Textbook

17 Consider the following statements.


1. If a sitting Rajya Sabha member contests and wins a Lok Sabha election, his seat in the Upper
House becomes automatically vacant on the date he is declared elected to Lok Sabha.
2. An individual can contest from two parliamentary constituencies but, if elected from both, he has to
resign one seat within 14 days of the declaration of the result, failing which both his seats shall fall
vacant.

Select the correct answer using the codes below.


A. 1 only
B. 2 only
C. Both 1 and 2
D. None of the above

Correct Answer : C

Answer Justification :

Justification: If a person is elected simultaneously to both Rajya Sabha and Lok Sabha, and if he
has not yet taken his seat in either House, he can choose, within 10 days from the later of the dates
on which he is chosen to those Houses, the House of which he would like to be a member.

[Article 101(1) of the Constitution read with Section 68(1) of The Representation of the People Act,

13
Total Marks : 200
Online Prelims TEST - 26 (TEXTBOOK)
( InsightsIAS Mock Test Series for UPSC Preliminary Exam 2020 ) Mark Scored : 0

1951] The member must intimate his choice in writing to the Secretary to the Election Commission
of India (ECI) within the 10-day window, failing which his seat in Rajya Sabha will fall vacant at the
end of this period. [Sec 68(2), RPA 1951].

The choice, once intimated, is final. [Sec 68(3), RPA, 1951] No such option is, however, available to
a person who is already a member of one House and has contested the election for membership of
the other House. So, if a sitting Rajya Sabha member contests and wins a Lok Sabha election, his
seat in the Upper House becomes automatically vacant on the date he is declared elected to Lok
Sabha.

The same applies to a Lok Sabha member who contests an election to Rajya Sabha. [Sec 69 read
with Sec 67A, RPA 1951] Elected on two Lok Sabha seats: There is no one in this category in the
new Lok Sabha. Under Sec 33(7) of RPA, 1951, an individual can contest from two parliamentary
constituencies but, if elected from both, he has to resign one seat within 14 days of the declaration
of the result, failing which both his seats shall fall vacant. [Sec 70, RPA, 1951 read with Rule 91 of
the Conduct of Elections Rules, 1961]

Q Source:
https://www.insightsonindia.com/wp-content/uploads/2019/07/InsightsonIndia-June-2019-Current-Af
fairs.pdf

18 As per the World Population Prospects 2019 report, India is projected to become the most populous
country by 2027 surpassing China, and host 1.64 billion people by 2050. The report was released by

A. World Economic Forum


B. United Nations
C. International Monetary Fund
D. Oxfam International

Correct Answer : B

Answer Justification :

Justification: World Population Prospects 2019 has been released by the Population Division of the
UN Department of Economic and Social Affairs. As per the report, India is projected to become the
most populous country by 2027 surpassing China, and host 1.64 billion people by 2050; the world as
a whole could be home to 8.5 billion people in just over a decade from now, and the number could
go up to 9.7 billion by mid-century.

Key findings: India specific: India will overtake China as the most populous country by around 2027.
India is also expected to add 273 million people by 2050 and will remain the most populated until
the end of the century. India leads the set of nine countries that will make up for more than half the
projected growth of the global population by 2050.

Top five: India is expected to remain the world’s most populous country with nearly 1.5 billion
inhabitants, followed by China at 1.1 billion, Nigeria with 733 million, the United States with 434
million, and Pakistan with an estimated population of 403 million.

14
Total Marks : 200
Online Prelims TEST - 26 (TEXTBOOK)
( InsightsIAS Mock Test Series for UPSC Preliminary Exam 2020 ) Mark Scored : 0

Q Source:
https://www.insightsonindia.com/wp-content/uploads/2019/07/InsightsonIndia-June-2019-Current-Af
fairs.pdf

19 Consider the following statements.


1. The Living Planet Report published by the World Wide Fund for Nature (WWF) is based on the
Living Planet Index (LPI) and ecological footprint calculations.
2. The LPI has been adopted by the Convention of Biological Diversity (CBD) as an indicator of
progress towards its 2011-2020 target to 'take effective and urgent action to halt the loss of
biodiversity'.

Which of the above is/are correct?


A. 1 only
B. 2 only
C. Both 1 and 2
D. None

Correct Answer : C

Answer Justification :

Justification: The Living Planet Report is published every two years by the World Wide Fund for
Nature since 1998.

It is based on the Living Planet Index and ecological footprint calculations.

According to recently released study, global wildlife populations have fallen by 58% since
1970 and if the trend continues then two-thirds of wild animals may go extinct by 2020.

The Living Planet Index (LPI) is a measure of the state of global biological diversity based on
population trends of vertebrate species from around the world. It does this in much the same
way that a stock market index tracks the value of a set of shares or a retail price index tracks
the cost of a basket of consumer goods.

The Living Planet Database (LPD) currently holds time-series data for over 20,000 populations
of more than 4,200 mammal, bird, fish, reptile and amphibian species from around the world,
which are gathered from a variety of sources such as journals, online databases and
government reports. Using a method developed by ZSL and WWF, these species population
trends are aggregated to produce indices of the state of biodiversity.

S2: The Living Planet Index (LPI) is a measure of the state of the world's biological diversity based
on population trends of vertebrate species from terrestrial, freshwater and marine habitats.

15
Total Marks : 200
Online Prelims TEST - 26 (TEXTBOOK)
( InsightsIAS Mock Test Series for UPSC Preliminary Exam 2020 ) Mark Scored : 0

The LPI played a pivotal role in measuring progress towards the 2010 target of the
Convention on Biological Diversity (CBD) to reduce the rate of biodiversity loss which,
according to the suite of 2010 biodiversity indicators, was not met.

In response, the 193 nations of the CBD committed themselves to a revised Strategic Plan for
Biodiversity, including the Aichi Biodiversity Targets , for the 2011-2020 period including
actions which will “…take effective and urgent action to halt the loss of biodiversity in order
to ensure that by 2020 ecosystems are resilient and continue to provide essential services,
thereby securing the planets variety of life, and contributing to human well-being, and poverty
eradication”.

The 2020 targets focus on maintaining ecosystem services in which biodiversity plays an
important role. The Living Planet Index is an applicable indicator for many of the Aichi
Biodiversity targets under Strategic Goals A-D which address the causes, pressures, state and
benefits of biodiversity.

Q Source: Important International Reports

20 Shanghai Cooperation Organisation (SCO) member states include


1. Afghanistan
2. Belarus
3. Mongolia
4. Iran

Select the correct answer using the codes below.


A. 2 and 3 only
B. 1 and 3 only
C. 1, 2 and 4 only
D. None of the above

Correct Answer : D

Answer Justification :

Justification: SCO comprises eight member states, namely the Republic of India, the Republic of
Kazakhstan, the People’s Republic of China, the Kyrgyz Republic, the Islamic Republic of Pakistan,
the Russian Federation, the Republic of Tajikistan, and the Republic of Uzbekistan;. The SCO counts
four observer states, namely the Islamic Republic of Afghanistan, the Republic of Belarus, the
Islamic Republic of Iran and the Republic of Mongolia. The SCO has six dialogue partners, namely
the Republic of Azerbaijan, the Republic of Armenia, the Kingdom of Cambodia, the Federal
Democratic Republic of Nepal, the Republic of Turkey, and the Democratic Socialist Republic of Sri
Lanka.

Learning: It is a Eurasian political, economic, and military organisation which was founded in 2001

16
Total Marks : 200
Online Prelims TEST - 26 (TEXTBOOK)
( InsightsIAS Mock Test Series for UPSC Preliminary Exam 2020 ) Mark Scored : 0

in Shanghai. Founding members: China, Kazakhstan, Kyrgyzstan, Russia, Tajikistan, and


Uzbekistan. The cooperation was renamed to Shanghai Cooperation Organisation after Uzbekistan
joined the organisation in 2001.

The SCO’s main goals are: strengthening mutual trust and neighbourliness among the member
states; promoting their effective cooperation in politics, trade, the economy, research, technology
and culture, as well as in education, energy, transport, tourism, environmental protection, and other
areas; making joint efforts to maintain and ensure peace, security and stability in the region; and
moving towards the establishment of a democratic, fair and rational new international political and
economic order. Presently, the

Q Source: International Organizations

21 The One Health concept, sometimes seen in news, recognizes

A. Inter-dependence between the policies of the member nations of the World Health
Organization (WHO)
B. Inter-connectivity among human health, the health of animals, and the environment
C. Inter-connectivity between landscape health and the health of flora and fauna
D. Inter-dependence of the flora for their mutual survival

Correct Answer : B

Answer Justification :

Justification: The World Organization of Animal Health, commonly known as OIE (an abbreviation
of its French title), summarises the One Health concept as “human health and animal health are
interdependent and bound to the health of the ecosystems in which they exist”.

The philosophy of One Health recognises inter-connectivity among human health, the health of
animals, and the environment.

Circa 400 BC, Hippocrates in his treatise On Airs, Waters and Places had urged physicians that all
aspects of patients’ lives need to be considered including their environment; disease was a result of
imbalance between man and environment. So One Health is not a new concept, though it is of late
that it has been formalised in health governance systems.

As human populations expand, it results in greater contact with domestic and wild animals,
providing more opportunities for diseases to pass from one to the other. Climate change,
deforestation and intensive farming further disrupt environment characteristics, while increased
trade and travel result in closer and more frequent interaction, thus increasing the possibility of
transmission of diseases.

Q Source:
https://www.insightsonindia.com/wp-content/uploads/2019/07/InsightsonIndia-June-2019-Current-Af
fairs.pdf

17
Total Marks : 200
Online Prelims TEST - 26 (TEXTBOOK)
( InsightsIAS Mock Test Series for UPSC Preliminary Exam 2020 ) Mark Scored : 0

22 “Deliverance Day” was celebrated in British India in the wake of

A. Passage of the Government of India Act 1935


B. Congress Ministries resigning from the Provinces in 1939
C. Khilafat agitation movement in 1919
D. Boycott of the Swadeshi movement

Correct Answer : B

Answer Justification :

Justification: An elimination based question; rest of the options pertain to events after or before
1939.

In 1937 elections were held under the provisions of the Government of India Act of 1935. Congress
Ministries were formed in seven states of India. In 1939 the Second World War broke out.

The British Government without consulting the people of India involved the country in the war. The
Congress vehemently opposed it and as a mark of protest the Congress Ministries in the Provinces
resigned on 12 December 1939.

The Muslim League celebrated that day as the Deliverance Day. In March 1940 the Muslim League
demanded the creation of Pakistan.

Q Source: 12th TN History Textbook

23 Consider the following statements.


1. The term ‘Sowa Rigpa’ is derived from Sanskrit language which means ‘Knowledge of Healing’.
2. The principle medical text of Sowa Rigpa that discusses some of its fundamental principles comes
from Tantra.

Select the correct answer using the codes below.


A. 1 only
B. 2 only
C. Both 1 and 2
D. None of the above

Correct Answer : C

Answer Justification :

Justification: Statement 1: Sowa-Rigpa” is one of the oldest, living and well documented medical
tradition popular in the world. The term ‘Sowa Rigpa’ is derived from Bhoti language which means
‘Knowledge of Healing’. It is an ancient Indian medical system which was enriched in the entire
Trans-Himalayan region. At present Sowa-Rigpa is more popular in Himalayan societies especially
in J & K region’, Ladakh ,Himachal Pradesh (Lahoul & Spiti), West Bengal (Darjeeling), Sikkim and

18
Total Marks : 200
Online Prelims TEST - 26 (TEXTBOOK)
( InsightsIAS Mock Test Series for UPSC Preliminary Exam 2020 ) Mark Scored : 0

Arunachal Pradesh and other parts of India. It is also being practiced in countries like Bhutan,
Mongolia and Russia etc.

Sowa Rigpa is formally recognized and promoted as traditional medical system by the Government
of India. Along with spread of Buddhism, Sowa-Rigpa also spread to other neighouring countries.

Statement 2: It can be perceived to be more close or having similarity with Ayurvedic philosophy/
principles of India since many of texts of Sowa-Rigpa are taken from one of the most famous treatise
of Ayurveda i.e. “Ashtanga Hridya” in one or other form.

Many (more than 75%) medicines used in Ayurveda (Indian origin) viz Triphala, Ashok,
Ashwagandha, Guggulu, Haridra etc. are also frequently used in Sowa- Rigpa System of Medicine
for treatment purposes.

The principle medical text “rGyud-bZi” Chatush Tantra-a textbook of fundamental principles of
Sowa-Rigpa is in Sanskrit language which was further translated into Bhoti language around 8th –
12th Century and further amended by Yuthok YontanGombo and other scholars of Trans Himalayan
region according to the socio-climatic conditions.

Q Source: http://ayush.gov.in/about-the-systems/sowa-rigpa/introduction-sowa-rigpa

24 In the context of Ancient India, masattuvan were

A. Messengers of peace sent by Indian kingdoms abroad


B. Bonded labourers who are admonished by the society
C. Successful merchants of south India
D. Closest relatives of the Kings who could be offered the Royal Throne

Correct Answer : C

Answer Justification :

Justification: A number of traders earned their living by sea-routes. Sea-faring was risk hut highly
profitable as it was very cheap.

The traders who took full advantage of this and became enormously rich were often
designated as masattuvan in Tamil and as setthis and satthavahas in Prakrit.

Learning: Some Indian commodities such as black-pepper, spices, medicinal herbs and silken and
cotton textiles etc. were in high demand in Rome. All these commodities were transported across
the Arabian Sea to the Mediterranean India had also close trading relations with Burma.

Q Source: Based on past year papers of UPSC

25 Consider the following about the National Medicinal Plants Board (NMPB).

19
Total Marks : 200
Online Prelims TEST - 26 (TEXTBOOK)
( InsightsIAS Mock Test Series for UPSC Preliminary Exam 2020 ) Mark Scored : 0

1. NMBP focusses on in-situ and ex-situ conservation and augmenting local medicinal plants and
aromatic species of medical significance.
2. NMPB seeks to support programs for quality assurance and standardization through development of
Good Agricultural and Collection Practices (GACPs).
3. Currently the board is working under the Ministry of AYUSH (Ayurveda, Yoga & Naturopathy,
Unani, Siddha & Homoeopathy), Government of India.

Select the correct answer using the codes below.


A. 1 only
B. 2 and 3 only
C. 1 and 2 only
D. 1, 2 and 3

Correct Answer : D

Answer Justification :

Background: India is one of the richest countries in the world in terms of biodiversity, has 15 agro-
climatic zones. Out of the 17000-18000 species of flowering plants, more than 7000 are estimated
to have medicinal usage in folk and documented systems of medicine like Ayurveda, Unani, Siddha
& Homoeopathy (AYUSH System of Medicine).

Justification: Statement 1: In order to promote medicinal plants sector, Government of India set
up National Medicinal Plants Board (NMPB) in the year 2000.

Currently the board is working under the Ministry of AYUSH (Ayurveda, Yoga & Naturopathy,
Unani, Siddha & Homoeopathy), Government of India. The primary mandate of NMPB is to develop
an appropriate mechanism for coordination between various ministries/ departments/ organization
and implementation of support policies/programs for overall (conservation, cultivation, trade and
export) growth of medicinal plants sector both at the Central /State and International level.

Aims & Objectives of the Board

In recent years cultivation of medicinal plants has started gaining momentum, still a
significant part of our requirements continue to be met from wild sources. To meet increasing
demand for medicinal plants NMBP focusses on in-situ & ex-situ conservation and augmenting
local medicinal plants and aromatic species of medical significance.

The NMPB also promote research & development, capacity building through trainings, raising
awareness through promotional activities like creation of Home/School herbal gardens.

NMPB also seek to support programs for quality assurance and standardization through
development of Good Agricultural and Collection Practices (GACPs), development of
monographs laying down standards of quality, safety and efficacy; development of agro-
techniques and credible institution a mechanism for certification of quality of raw drugs,

20
Total Marks : 200
Online Prelims TEST - 26 (TEXTBOOK)
( InsightsIAS Mock Test Series for UPSC Preliminary Exam 2020 ) Mark Scored : 0

seeds and planting material.

Overall, NMPB’s main objective is the development of medicinal plants sector through
developing a strong coordination between various ministries/ departments/ organization for
implementation of policies / programs on medicinal plants.

Q Source: https://nmpb.nic.in/about-us

26 Consider the following statements.


1. Plastic Park is an industrial zone developed in cluster approach for establishing units for plastic
enterprises and its allied industries.
2. Plastic Park are being developed by State Governments with the grant-in-aid funding support from
Department of Chemicals & Petrochemicals, Ministry of Chemicals & Fertilizers.

Select the correct answer using the codes below.


A. 1 only
B. 2 only
C. Both 1 and 2
D. None of the above

Correct Answer : C

Answer Justification :

Justification: Plastic Park is an industrial zone, developed in cluster approach for establishing
units for plastic enterprises and its allied. Industries to consolidate and synergize the capacities of
the domestic downstream plastic processing industry. Plastic Park has state-of-the-art
infrastructure with common facilities including plastic waste management system.

Plastic Parks plays key role in growth of the plastic processing community material and machinery
suppliers, plastic processing companies, plastic recycling companies.

The Plastic Parks are being developed by State Governments with the grant-in-aid funding support
from Department of Chemicals & Petrochemicals, Ministry of Chemicals & Fertilizers. These parks
have great potential for attracting investments and generating more employment opportunities for
the local population.

Q Source: https://chemicals.nic.in/plastic-parks

27 Consider the following about certain anthropogenic sources of pollution.


1. Coal-fired power plants emit mercury and dioxins.
2. Mines engaged in gold production release arsenic and lead.
3. Cement production releases sulfur dioxide and nitrogen dioxide.
4. Manufacture of batteries releases Mercury.

21
Total Marks : 200
Online Prelims TEST - 26 (TEXTBOOK)
( InsightsIAS Mock Test Series for UPSC Preliminary Exam 2020 ) Mark Scored : 0

Select the correct answer using the codes below.


A. 1, 2 and 3 only
B. 2 and 4 only
C. 1 only
D. 1, 2, 3 and 4

Correct Answer : D

Answer Justification :

Justification: Statement 1: These plants emit harmful pollutants, including mercury, non-mercury
metallic toxics, acid gases, and organic air toxics such as dioxin.

Statement 2: Toxic gold mine waste includes dangerous chemicals such as arsenic, lead, mercury,
petroleum byproducts, acids, cyanide etc.

Statement 3: The cement sector is one of the largest industrial sources of pollution, emitting
pollutants like mercury and harmful gases like sulfur dioxide, nitrogen oxide, and carbon monoxide.

Statement 4: We have covered this in an earlier test discussing Li-ion batteries.

Q Source: AR: Past year UPSC papers

28 The three largest states in terms of geographical area include

A. Rajasthan, Uttar Pradesh (UP) and Maharashtra


B. Rajasthan, Madhya Pradesh (MP) and UP
C. Madhya Pradesh, Maharashtra and Rajasthan
D. Rajasthan, MP and Maharashtra

Correct Answer : D

Answer Justification :

Learning: Below given is a list of cities and states population wise & area wise

22
Total Marks : 200
Online Prelims TEST - 26 (TEXTBOOK)
( InsightsIAS Mock Test Series for UPSC Preliminary Exam 2020 ) Mark Scored : 0

23
Total Marks : 200
Online Prelims TEST - 26 (TEXTBOOK)
( InsightsIAS Mock Test Series for UPSC Preliminary Exam 2020 ) Mark Scored : 0

Q Source: General Census based questions

29 Consider the following about the International Air Transport Association (IATA).
1. IATA sets mandatory environmental and emissions related goals for all its members and industry
stakeholders.
2. IATA fixes an airline tariff cap for its member nations, pricing beyond which entails penalty for the
airline.

Select the correct answer using the codes below.


A. 1 only
B. 2 only
C. Both 1 and 2
D. None of the above

Correct Answer : D

Answer Justification :

Justification: The International Air Transport Association (IATA) is a trade association of the

24
Total Marks : 200
Online Prelims TEST - 26 (TEXTBOOK)
( InsightsIAS Mock Test Series for UPSC Preliminary Exam 2020 ) Mark Scored : 0

world's airlines founded in 1945. IATA has been described as a cartel since, in addition to setting
technical standards for airline, IATA also organized tariff conferences that served as a forum for
price fixing. There are no mandatory price fixations.

Consisting of 290 airlines, primarily major carriers, representing 117 countries, the IATA's member
airlines account for carrying approximately 82% of total available seat miles air traffic.

IATA supports airline activity and helps formulate industry policy and standards. It is headquartered
in Canada in the city of Montréal, with Executive Offices in Geneva, Switzerland.

IATA provides consulting and training services in many areas.

A number of standards are defined under the umbrella of IATA. One of the most important is the
transport of dangerous goods (HAZMAT).

Statement 2: IATA members and all industry stakeholders have agreed to three sequential
environmental goals:

An average improvement in fuel efficiency of 1.5% per annum from 2009 through 2020

A cap on net carbon emissions from aviation from 2020 (carbon-neutral growth)

A 50% reduction in net aviation carbon emissions by 2050 relative to 2005 levels.

These goals are not mandatory and do not entail a penalty for violation.

Q Source: https://www.civilaviation.gov.in/: Organizations

30 Mammals cannot be

A. Endothermic
B. Invertebrates
C. Oviparous
D. Any of the above

Correct Answer : B

Answer Justification :

Justification: All mammals are:

Are vertebrates (which means they have a backbone or spine).

25
Total Marks : 200
Online Prelims TEST - 26 (TEXTBOOK)
( InsightsIAS Mock Test Series for UPSC Preliminary Exam 2020 ) Mark Scored : 0

Are endothermic. Also known as “warm-blooded,” endothermic animals regulate their own
body temperate which allows them to live in almost every climate on Earth.

Have hair on their bodies.

Produce milk to feed their babies.

All breathe through lungs.

Option C: Four species of Echidna and the Platypus are the only mammals which are oviparous; rest
are viviparous i.e. give birth to live young ones.

Q Source: AR: Past year UPSC papers

31 Consider the following about Telecommunications Dispute Settlement and Appellate Tribunal
(TDSAT).
1. The Chairperson should be or should have been a Judge of the High Court or Supreme Court.
2. TDSAT exercises original jurisdiction over cyber matters, that is, it is a court of first instance.
3. Jurisdiction of TDSAT extends to airport tariff matters.

Select the correct answer using the codes below.


A. 1, 2 and 3
B. 2 and 3 only
C. 2 only
D. 3 only

Correct Answer : D

Answer Justification :

Justification: In order to bring in functional clarity and strengthen the regulatory framework and
the disputes settlement mechanism in the telecommunication sector, the TRAI Act of 1997 was
amended in the year 2000 and TDSAT was set up to adjudicate disputes and dispose of appeals with
a view to protect the interests of service providers and consumers of the telecom sector and to
promote and ensure orderly growth of the telecom sector. In January 2004, the Government
included broadcasting and cable services also within the purview of TRAI Act. After coming into
force of the relevant provisions of the Finance Act 2017, the jurisdiction of TDSAT stands extended
to matters that lay before the Cyber Appellate Tribunal and also the Airport Economic Regulatory
Authority Appellate Tribunal.

Statement 1: Composition of TDSAT

The Tribunal consists of a Chairperson and two Members appointed by the Central Government.
The Chairperson should be or should have been a Judge of the Supreme Court or the Chief Justice of

26
Total Marks : 200
Online Prelims TEST - 26 (TEXTBOOK)
( InsightsIAS Mock Test Series for UPSC Preliminary Exam 2020 ) Mark Scored : 0

a High Court. A Member should have held the post of Secretary to the Government of India or any
equivalent post in the Central Government or the State Government for a period of not less than two
years or a person who is well versed in the field of technology, telecommunication, industry,
commerce or administration. Powers and Jurisdiction.

Statement 2: The Tribunal exercises jurisdiction over Telecom, Broadcasting, IT and Airport tariff
matters under the TRAI Act, 1997 (as amended), the Information Technology Act, 2008 and the
Airport Economic Regulatory Authority of India Act, 2008. The Tribunal exercises original as well as
appellate jurisdiction in regard to Telecom, Broadcasting and Airport tariff matters. In regard to
Cyber matters the Tribunal exercises only the appellate jurisdiction.

Statement 3: The Tribunal is the Court of first instance except cyber matters. ii. Every proceeding
before the Tribunal is deemed to be a judicial proceeding within the meaning of sections 193 and
228, and for the purposes of section 196, of the Indian Penal Code (45 of 1860).

The Tribunal is deemed to be a civil court for the purposes of section 195 and Chapter XXVI of the
Code of Criminal Procedure, 1973 (2 of 1974).

Tribunal’s Orders are executable as a decree of civil court.

Q Source: http://www.tdsat.gov.in/admin/introduction/uploads/TDSAT%20INTRO.pdf

32 In the Indian Parliamentary model of Government

A. The Executive is responsible to none but Constitutional ideals


B. The Legislature controls the Judiciary
C. The Executive is responsible to the Legislature
D. The Judiciary controls the Executive

Correct Answer : C

Answer Justification :

Learning: The relationship between the executive and the legislature in a parliamentary system is
called responsible government.

Here the executive branch derives its democratic legitimacy from its ability to command the
confidence of the legislative branch. Legislature also holds the executive accountable.

Q Source: Basic of Indian Polity

33 Consider the following about Centre for Development of Telematics (C-DOT).


1. It is a registered ‘public funded research institution’ with the Department of Scientific and
Industrial Research (DSIR).
2. It works based on the Technology Transfer model to facilitate the bulk production of high-quality
Telecom products and solutions by telecom manufacturers.

27
Total Marks : 200
Online Prelims TEST - 26 (TEXTBOOK)
( InsightsIAS Mock Test Series for UPSC Preliminary Exam 2020 ) Mark Scored : 0

Select the correct answer using the codes below.


A. 1 only
B. 2 only
C. Both 1 and 2
D. None of the above

Correct Answer : C

Answer Justification :

Justification: Centre for Development of Telematics (C-DOT) was established in August 1984 as an
autonomous Telecom R&D centre of DoT, Govt. of India. It is a registered society under the
Societies Registration Act, 1860. It is a registered ‘public funded research institution’ with the
Department of Scientific and Industrial Research (DSIR), Ministry of Science & Technology,
Government of India.

Hailed as the progenitor of the Indigenous Telecom Revolution in the nation, C-DOT, with more than
3 decades of its relentless R&D efforts in the indigenous design, development and production of
telecom technologies especially suited to the Indian landscape, have been in the technology
forefront and has significantly contributed to the Indian Telecom Network’s Digitization.

In the initial years, C-DOT triggered a telecom revolution in the rural India that was responsible for
all-round socio-economic development. As part of its development process, C-DOT has spawned a
wide base of equipment manufacturers and component vendors for the industry. Moreover, it has
been instrumental in the evolution of a Telecom Manufacturing Ecosystem based on its Technology
Transfer model to facilitate manufacturers in the bulk production of high-quality Telecom products
and solutions.

Learning: C-DOT, over the years, has evolved into a full-fledged telecom R&D institution, that
complies with level-5 maturity on CMMI Model, and has capabilities to undertake large-scale state-
of-the-art telecom technologies development programs. C-DOT’s commitment to the cause of nation
building continues with development of products of national and strategic importance. C-DOT as a
torch bearer of indigenous telecom R&D continues to develop latest technology products in areas
like Optical, Switching, Wireless, Security and Network Management while also working on
futuristic technologies like M2M/IOT, 5G, AI, etc.

Q Source: http://www.cdot.in/cdotweb/web/aboutus.php?lang=en#corporate_profile

34 In India, most freight traffic is presently handled by the

A. Roadways
B. Railways
C. Inland waterways
D. Airways

Correct Answer : A

28
Total Marks : 200
Online Prelims TEST - 26 (TEXTBOOK)
( InsightsIAS Mock Test Series for UPSC Preliminary Exam 2020 ) Mark Scored : 0

Answer Justification :

Justification: Network expansion in Railways has lagged behind capacity addition in the domestic
roads sector. So, significant freight share has gone to the roadways (57% of total), instead of more
efficient Railways (36% only).

Inland waterways and coastal shipping are catching up with shares of less than 10%.

Learning: One of the reasons why railways, despite its humungous size, remains behind in freight
traffic (with poor growth) is because railways are under-invested. It accounts for only 5.5% of total
plan outlay vis-a-vis 11% in other transport sectors. China has invested 11 times more per capita
than in India.

In 1990s China lagged behind India, today it is way ahead.

In USA, freight share of railways is 44% despite having extensive coastal shipping and inland
waterways linkages.

Q Source: Chapter on Trade: NCERT 12th: India, People and Economy

35 The World Press Freedom Prize is conferred by

A. Reporters without Borders


B. UNESCO
C. Amnesty International
D. International Federation of Journalists

Correct Answer : B

Answer Justification :

Justification: The World Press Freedom Prize also known as UNESCO/Guillermo Cano World Press
Freedom Prize is formally conferred every year by Director-General of UNESCO, on occasion of
World Press Freedom Day observed on 3 May.

UNESCO/Guillermo Cano World Press Freedom Prize: Created in 1997, the annual
UNESCO/Guillermo Cano World Press Freedom Prize honours a person, organization or institution
that has made an outstanding contribution to the defence and, or promotion of press freedom
anywhere in the world, and especially when this has been achieved in the face of danger.

Q Source: In news

36 Consider the following statements related to the Central fund allocations to states.
1. The Centrally ‘Sponsored’ Schemes are funded and implemented in states exclusively by the Central
government.
2. Assistance under Centrally ‘Sponsored’ Schemes is extended by the Central Government to States
under Article 282 of the Constitution.

29
Total Marks : 200
Online Prelims TEST - 26 (TEXTBOOK)
( InsightsIAS Mock Test Series for UPSC Preliminary Exam 2020 ) Mark Scored : 0

Select the correct answer using the codes below.


A. 1 only
B. 2 only
C. Both 1 and 2
D. None of the above

Correct Answer : B

Answer Justification :

Justification: Statement 1: Central sector schemes (not Centrally sponsored schemes) are 100%
funded by the Union government and implemented by the Central Government or/and the State
machinery. Central sector schemes are mainly formulated on subjects from the Union List.

Statement 2: Centrally Sponsored Schemes (CSS) are schemes that are implemented by state
governments but are largely funded by the Central Government with a defined State Government
share.

They are basically special purpose grants (or loans) extended by Central Government to states
to encourage them to plan and implement programmes that help attain national goals and
objectives.

CSS are basically extended by the Central Government to States under Article 282 of the
Constitution. It mainly covers items listed in states list.

Q Source: Schemes and Programmes of Government of India

37 Taylor’s Rule, sometimes seen in news, is related to

A. Power consumption
B. Gross Capital formation efficiency
C. Administrative efficiency
D. Monetary policy rates

Correct Answer : D

Answer Justification :

Learning: It is a highly acclaimed rule for monetary policy. It says that the central bank should
increase interest rates by a margin slightly exceeding the current inflation rate to stabilise GDP.

The Taylor Rule was formulated through a post facto analysis of US Fed monetary policy,
culminating in a statistical relationship that best fitted past successful monetary policy.

30
Total Marks : 200
Online Prelims TEST - 26 (TEXTBOOK)
( InsightsIAS Mock Test Series for UPSC Preliminary Exam 2020 ) Mark Scored : 0

Interest rate = inflation rate +0.5*(actual GDP minus potential GDP) +0.5*(inflation rate minus 2)+
2

The rule states that when inflation goes up 1 point above the target, the Central Bank should
counteract the increase by raising interest rates by 1 .5 points.

By increasing nominal interests by more than the increase in inflation, Taylor’s rule increases
real interest rates—cooling oil the economy—when inflation increases.

Q Source: Indian Economy: Ramesh Singh

38 The Parliament exercises Budgetary control over the executive, that is, control ‘before’ the
appropriation of grants through the enactment of the budget by

A. Estimate Committee
B. Departmental Standing Committees
C. Public Accounts Committee
D. Committee on Public Undertaking

Correct Answer : B

Answer Justification :

Learning: The parliamentary control over the Executive in financial matters operates in two stages:

Budgetary control, that is, control before the appropriation of grants through the enactment
of the budget; and

Post-budgetary control, that is, control after the appropriation of grants through the three
financial committees – PAC, Estimate Committee and CoPU.

Option B: The Departmental standing committees consider the demands for grants of the concerned
ministries /departments before they are discussed and voted in the Lok Sabha. It also considers the
annual reports of the Ministries and departments.

Option C: The function of the PAC is to examine the annual audit reports of the CAG, which are laid
before the Parliament by the president. This is a kind of post-mortem of the executive expenditures.

Q Source: Chapter on Parliament: Indian Polity: M Laxmikanth

39 Consider the following statements.


1. Microbiome is the community of microorganisms that live in the human body.

31
Total Marks : 200
Online Prelims TEST - 26 (TEXTBOOK)
( InsightsIAS Mock Test Series for UPSC Preliminary Exam 2020 ) Mark Scored : 0

2. Microbiomes start forming at birth and their condition can help predict future health.

Select the correct answer using the codes below.


A. 1 only
B. 2 only
C. Both 1 and 2
D. None of the above

Correct Answer : C

Answer Justification :

Justification: Microbiome is the community of bacteria, viruses and fungi that live on the skin or in
the gut, nose or reproductive tract.

Microbiomes start forming at birth and are different depending on whether babies were born
vaginally or via C-section.

And they change with age and different exposures, such as a course of antibiotics that can wipe out
friendly bacteria along with infection-causing ones. Microbes in body could help predict future
health.

A study found microbial zoo changing in ways that eventually may help doctors determine who’s at
risk of preterm birth, inflammatory bowel disease, even diabetes.

Q Source: Basics: Biology: Based on past year UPSC papers

40 Consider the following statements.


1. The Satavahanas started the practice of granting tax-free villages to brahmanas and Buddhist
monks.
2. The terms ‘kataka’ and ‘skandhavaras’ in satvahana related manuscripts refer to land grants.

Which of the above is/are correct?


A. 1 only
B. 2 only
C. Both 1 and 2
D. None

Correct Answer : A

Answer Justification :

Justification: Statement 1: The Satavahanas started the practice of granting tax-free villages to
brahmanas and Buddhist monks.

32
Total Marks : 200
Online Prelims TEST - 26 (TEXTBOOK)
( InsightsIAS Mock Test Series for UPSC Preliminary Exam 2020 ) Mark Scored : 0

The cultivated fields and villages granted to them were declared free from molestation by
royal policemen and soldiers, and all kinds of royal officers.

These areas therefore became small independent islands within the Satavahana kingdom.

Possibly the Buddhist monks also preached peace and rules of good conduct among the
people they lived with, and taught them to respect political authority and social order.

Statement 2: The military character of the Satavahana rule is evident from the common use of such
terms as kataka and skandhavaras in their inscriptions. These were military camps and settlements
which served as administrative centres so long as the king was there. Thus coercion played an
important part in the Satavahana administration.

Q Source: Based on past year papers

41 Which of these nations has an observer status at the South Asian Association for Regional
Cooperation (SAARC)?

A. China
B. Afghanistan
C. United Kingdom
D. South Africa

Correct Answer : A

Answer Justification :

Learning: States with observer status include Australia, China, the European Union, Iran, Japan,
Mauritius, Myanmar, South Korea and the United States.

Myanmar has expressed interest in upgrading its status from an observer to a full member of
SAARC. Russia has applied for observer status membership of SAARC. Turkey applied for observer
status membership of SAARC in 2012. South Africa has participated in meetings.

Q Source: International and Regional Organizations

42 Consider the following statements.


Assertion (A): The Chief Election Commissioner (CEC) can be overruled by the two Election
Commissioner (ECs).
Reason (R): The opinion of each election commissioner of the Election Commission of India (ECI)
carries equal weight.

In the context of the above, which of these is correct?


A. A is correct, and R is an appropriate explanation of A.

33
Total Marks : 200
Online Prelims TEST - 26 (TEXTBOOK)
( InsightsIAS Mock Test Series for UPSC Preliminary Exam 2020 ) Mark Scored : 0

B. A is correct, but R is not an appropriate explanation of A.


C. A is correct, but R is incorrect.
D. Both A and R are incorrect.

Correct Answer : A

Answer Justification :

Justification: If some difference of opinion persists even after oral deliberations and discussions,
such dissent is recorded in the file.

All opinions carry equal weight, which means the CEC can be overruled by the two ECs. In normal
practice, while communicating the decision of the Commission in executive matters, the majority
view is conveyed to the parties concerned.

The dissent remains recorded in the file. In case dissent is to be recorded in a case of judicative
nature, the dissenting member may like to record a separate opinion/order

Q Source: Basics: Polity: Revision

43 Bahishkrit Hitakarini Sabha was the first organization formed by

A. Atmaram Pandurang
B. Motilal Nehru
C. B. R. Ambedkar
D. Mahadev Govind Ranade

Correct Answer : C

Answer Justification :

Justification: Bahishkrit Hitakarini Sabha was the first organization formed by Dr. Ambedkar in
1924. Ambedkar was appointed as the first Law Minister of Independent India, but he resigned from
the Cabinet in 1951 due to differences with Nehru on the Hindu Code Bill.

Ambedkar got himself converted to Buddhism in 1956. After his death, his political party
Scheduled Caste Federation was renamed as Republican Party of India in 1957 by his
followers.

Some of his famous books are: ‘The Untouchable: Who are They and Why They Have Become
Untouchables’; ‘Buddha and His Dhamma’; ‘The Rise and Fall of Hindu Women‘,
‘Emancipation of Untouchables’, ‘The Evolution of Provincial Finance in British India’;
‘Pakistan or Partition of India‘, ‘Thoughts on Linguistic States’, etc.

34
Total Marks : 200
Online Prelims TEST - 26 (TEXTBOOK)
( InsightsIAS Mock Test Series for UPSC Preliminary Exam 2020 ) Mark Scored : 0

Q Source: 10th NCERT: Contemporary India - II

44 Rule 49MA, under the Conduct of Elections Rules, 1961, framed by the Election Commission of India
was recently in news with regards to

A. Asset declaration of candidates with criminal background


B. Plea for revisiting election results in the Supreme Court
C. False complaint of malfunction of an electronic voting machine
D. Registration of regional political parties

Correct Answer : C

Answer Justification :

Justification: Rule 49MA is mentioned under ‘The Conduct of Elections Rules’ . Under the rule,
where printer for paper trail is used, if an elector after having recorded his vote under rule 49M
alleges that the paper slip generated by the printer has shown the name or symbol of a candidate
other than the one he voted for, the presiding officer shall obtain a written declaration from the
elector as to the allegation, after warning the elector about the consequence of making a false
declaration.

The rules outline that if after investigation, the allegation of EVM malfunctioning is found to be
false or incorrect, then the complainant can be prosecuted under Section 177 of the Indian Penal
Code for “furnishing false information”. In such a case, a jail term of six months or a fine of Rs
1,000 or both is guaranteed.

The Election Commission may “revisit” the rule for prosecution of a voter for making a false
complaint of malfunction of an electronic voting machine or a voter verifiable paper audit trail
machine- Rule 49MA.

The Supreme Court, in April 2019, sought a response from the Election Commission on a plea
seeking abolition of a provision in election rules that provides for prosecution of an elector if a
complaint alleging malfunctioning of EVMs and VVPATs turns out to be false.

Q Source: In news

45 The first Indian woman to hold a cabinet post (India) and serve as the first woman President of the
United Nations General Assembly is

A. Vijaya Laxmi Pandit


B. Rajkumari Amrit Kaur
C. Sarojini Naidu
D. Aruna Asaf Ali

Correct Answer : A

35
Total Marks : 200
Online Prelims TEST - 26 (TEXTBOOK)
( InsightsIAS Mock Test Series for UPSC Preliminary Exam 2020 ) Mark Scored : 0

Answer Justification :

Learning: In 1937 she was elected to the provincial legislature of the United Provinces and was
designated minister of local self-government and public health.

She held the latter post until 1939 and again from 1946 to 1947.

In 1946 she was elected to the Constituent Assembly from the United Provinces.

Following India's freedom from British occupation in 1947 she entered the diplomatic service
and became India's ambassador to the Soviet Union from 1947 to 1949

Between 1946 and 1968, she headed the Indian delegation to the United Nations and in 1953
served as the first woman President of the United Nations General Assembly.

Q Source: AR: Past year UPSC papers

46 Consider the following statements.


1. The Bonn Challenge is a global effort to bring 150 billion hectares of deforested and degraded land
into restoration by 2020.
2. India has joined the voluntary Bonn Challenge pledge to bring into restoration several million
hectares of degraded and deforested land by the year 2020.

Select the correct answer using the codes below.


A. 1 only
B. 2 only
C. Both 1 and 2 only
D. None of the above

Correct Answer : B

Answer Justification :

Justification: At the UNFCC Conference of the Parties (COP) 2015 in Paris, India also joined the
voluntary Bonn Challenge pledge to bring into restoration 13 million hectares of degraded and
deforested land by the year 2020, and additional 8 million hectares by 2030. India’s pledge is one of
the largest in Asia.

The Bonn Challenge is a global effort to bring 150 million hectares of deforested and degraded land
into restoration by 2020 and 350 million hectares by 2030. The 2020 target was launched at a high
level event in Bonn in 2011 organised by the Government of Germany and IUCN, and was later
endorsed and extended to 2030 by the New York Declaration on Forests of the 2014UN Climate
Summit.

36
Total Marks : 200
Online Prelims TEST - 26 (TEXTBOOK)
( InsightsIAS Mock Test Series for UPSC Preliminary Exam 2020 ) Mark Scored : 0

The Bonn Challenge is an implementation vehicle for national priorities such as water and food
security and rural development while simultaneously helping countries contribute to the
achievement of international climate change, biodiversity and land degradation commitments.

Underlying the Bonn Challenge is the forest landscape restoration (FLR) approach, which aims to
restore ecological integrity at the same time as improving human well-being through
multifunctional landscapes.

It will create approximately USD 84 billion per year in net benefits that could bring direct additional
income opportunities for rural communities

Learning: Forest landscape restoration (FLR) is the on-going process of regaining ecological
functionality and enhancing human well-being across deforested or degraded forest landscapes.
FLR is more than just planting trees – it is restoring a whole landscape to meet present and future
needs. It is long-term because it requires a multi-year vision of the ecological functions.

The majority of restoration opportunities are found on or adjacent to agricultural or pastoral land.
In these situations, restoration must complement and not displace existing land uses. This result in
a mosaic of different land uses including: agriculture, agroforestry systems and improved ecological
corridors.

It integrates a number of guiding principles, including: Focus on landscapes, restore functionality,


Involve stakeholders, Tailor to local conditions and Avoid further reduction of natural forest cover.

Q Source: Frequently in news

47 Consider the following about the Anuvrat Movement of Jainism.


1. It advocated that Dharma is not for ensuring happiness in the future lives but also for achieving
happiness in the present life.
2. Yuvacharya Mahapragya opposed Anuvratas and founded a new sect of Jainism.
3. It dictated application of anuvrata principles strictly to one’s religious life and not inter-mingle it
with one’s non-religious life.

Select the correct answer using the codes below.


A. 1 only
B. 2 and 3 only
C. 3 only
D. 2 only

Correct Answer : A

Answer Justification :

Justification: Statement 1: The movement held that Dharma is not for ensuring happiness in the
future lives but also for achieving happiness in the present life.

Statement 2: The movement continued under the leadership of his disciple Yuvacharya Mahapragya.

37
Total Marks : 200
Online Prelims TEST - 26 (TEXTBOOK)
( InsightsIAS Mock Test Series for UPSC Preliminary Exam 2020 ) Mark Scored : 0

He was raised to the rank of the Acharya by Acharya Tulsi himself.

Learning: The Anuvrat Movement was launched in 1949 (literal meaning is small vow, which is the
limited version of the Mahavratas for the monks).

It was based on the five Jain principles Truth, Nonviolence, Non-possession, Non-stealing and
Celibacy as applied in their limited version for the lay people.

Q Source: AR: Past year UPSC papers

48 Which of the following will reduce Global Warming in the short-term?

A. Increased rice cultivation


B. Greater promotion of cattle breeding in developing countries
C. Melting of permafrost In the Arctic region
D. Major and sustained volcanic explosions

Correct Answer : D

Answer Justification :

Justification: Option A: Rice fields are wetlands that release methane which is a GHG accelerating
global warming.

Option B: Cattle breeding also release methane. So, B is wrong.

Option C: Permafrost contains carbon that was trapped since ages, and its release after melting
accelerates global warming. The concept has been covered in an earlier test.

Option D: Suspended particles from volcanic ash, when spread in the atmosphere, reduce solar
insolation and help cool off the earth.

Q Source: Concepts related to Environment

49 The Kailasanatha temple at Ellora and the Shore Temple at Mamallapuram


1. are both rock-cut temples
2. are both located near shores

Select the correct answer using the codes below.


A. 1 only
B. 2 only
C. Both 1 and 2
D. None of the above

Correct Answer : D

38
Total Marks : 200
Online Prelims TEST - 26 (TEXTBOOK)
( InsightsIAS Mock Test Series for UPSC Preliminary Exam 2020 ) Mark Scored : 0

Answer Justification :

Justification: Kailasanatha is a rock cut temple, but Shore temple is not. Also, Ellora is not on the
coastline.

The Shore temple faces east towards the sea and has three shrines – east and west to Shiva and the
middle for Vishnu (Anantashayana).

Learning: Rajasimha Group introduced the structural temples and Gopura style in Pallava
architecture. The Kailasnath temple at Kanchi and the Shore temple at Mahabalipuram are
examples.

Q Source: AR: Past year UPSC papers

50 The Social & Labour Convergence Program (SLCP) is an initiative led by the world's leading brands,
industry groups and civil society organizations. SLCP is being launched targeting which of these
industries/sectors in India currently?

A. Forest produce
B. Textiles
C. Sugarcane farming and processing
D. Information Technology

Correct Answer : B

Answer Justification :

Justification: The Social & Labour Convergence Program (SLCP) is an initiative led by the world's
leading manufacturers, brands, retailers, industry groups, (inter)governmental organizations,
service providers and civil society organizations, to eliminate audit fatigue by replacing current
proprietary tools with a standard-neutral Converged Assessment Framework.

The mission of the SLCP is to improve working conditions by allowing resources that were
previously designated for compliance audits to be redirected towards the improvement of social and
labour conditions. In the first roll-out phase in late-May 2019, SLCP will launch operations in India
as well as China, Sri Lanka and Taiwan.

The benefits of SLCP for facilities are that it addresses audit fatigue by reducing the number of
social audits and facilitates measuring of employment practices, thus improving working conditions
& employee relations. It also redeploys resources towards improvement actions and fosters trust
and collaboration between supply chain partners. SLCP will be holding a series of free one-day
seminars in four centres to introduce facilities and their business partners to the SLCP process.

The cotton textiles export promotion council popularly known as Texprocil is organising the launch
event for SLCP in Mumbai on 5th June and Dr K V Srinivasan, Chairman Texprocil welcomed the
launch of the SLCP in India at various other centres also like Delhi, Bengaluru, and Tirupur.

"Issues of social and labour compliance become highly relevant in industries which are labour-

39
Total Marks : 200
Online Prelims TEST - 26 (TEXTBOOK)
( InsightsIAS Mock Test Series for UPSC Preliminary Exam 2020 ) Mark Scored : 0

intensive and the textile and clothing industry is one such sector", said Dr K V Srinivasan.

Q Source:
https://www.business-standard.com/article/news-ani/social-labour-convergence-program-slcp-launch
ing-operations-in-india-119052401270_1.html

51 The salient features and the aims of Delhi Declaration on Traditional Medicines for South East Asian
countries include
1. To develop institutionalized mechanism for exchange of information, expertise and knowledge with
active cooperation with WHO on traditional medicine
2. To promote National policies, strategies and interventions for equitable development and
appropriate use of traditional medicine in the health care delivery system

Select the correct answer using the codes below.


A. 1 only
B. 2 only
C. Both 1 and 2
D. None of the above

Correct Answer : C

Answer Justification :

Background: Almost every country in the world has its own form of traditional medicine and
demand globally for it is increasing. WHO has launched its new Traditional Medicine Strategy
2014–2023, which aims to harness the potential contribution of traditional medicine to health,
wellness, people-centred health care and universal health coverage.

The Strategy also aims to promote best practices in the use of safe and good-quality traditional
medicine through the regulation, research and integration of traditional medicine products,
practices and practitioners into the health system, as appropriate.

In 2013, the health ministers of the Member States of the WHO South-East Asia Region signed the
Delhi Declaration.

Justification: Its salient features include:

I. To promote National policies, strategies and interventions for equitable development and
appropriate use of traditional medicine in the health care delivery system.

II. To develop institutionalized mechanism for exchange of information, expertise and knowledge
with active cooperation with WHO on traditional medicine through workshops, symposia, visit of
experts, exchange of literature etc.

III. To establish regional centres as required for capacity building and networking in the areas of
traditional medicine and medicinal plants.

The Ministry of AYUSH has already set up Ayurveda Chair in University of Debrecen, Hungary and

40
Total Marks : 200
Online Prelims TEST - 26 (TEXTBOOK)
( InsightsIAS Mock Test Series for UPSC Preliminary Exam 2020 ) Mark Scored : 0

Rangsit University, Thailand.

Q Source:
http://ayush.gov.in/international-cooperation/delhi-declaration-traditional-medicine-se-asian-countri
es

https://apps.who.int/iris/handle/10665/129404

52 Consider the following statements.


1. Bhutia tribe practice Tantric Buddhism.
2. Rengmas tribes discourage painting as a sign of bad omen.
3. Garo is the only tribe that does not celebrate harvest festivals.

Select the correct answer using the codes below.


A. 1 only
B. 2 and 3 only
C. 3 only
D. 1 and 2 only

Correct Answer : A

Answer Justification :

Justification: Statement 1: These are tribal community which belongs from Sikkim that have
migrated from Tibet. They livein rectangular shaped house called Khin and follows Tantric
Buddhism.

Statement 2: Rengmas are famous for making yellow dye from the flowers of a tree. They also
practice painting on cloth.

Statement 3: Wangala is one of their major festival which they celebrate as thanks giving ceremony
after harvesting of the crops. The important cultivation crops of the tribes are rice, ginger, millet
etc.

Q Source: Tribes of India

53 Consider the following about Half-an-hour discussions in Lok Sabha.


1. No prior notice is required for raising matters.
2. The speaker is bound to admit matters raised by members that seek to revise the policy of
Government.

Which of the above is/are correct?


A. 1 only
B. 2 only
C. Both 1 and 2
D. None

41
Total Marks : 200
Online Prelims TEST - 26 (TEXTBOOK)
( InsightsIAS Mock Test Series for UPSC Preliminary Exam 2020 ) Mark Scored : 0

Correct Answer : D

Answer Justification :

Justification: Statement 1: A member wishing to raise a matter shall give notice in writing to the
Secretary-General three days in advance of the day on which the matter is desired to be raised, and
shall shortly specify the point or points that he wishes to raise: Provided that the notice shall be
accompanied by an explanatory note stating the reasons for raising discussion on the matter in
question.

Statement 2: The speaker is not obliged to admit the motion if it focuses on changing an existing
government policy, since it no half-hour discussion motions are passed by the house and thus they
are admitted entirely on the request of the member.

Q Source: http://parliamentofindia.nic.in/ls/rules/rules.html

54 Arrange the following geological time scales chronologically (starting from the earliest in history):
1. Cenozoic
2. Mesozoic
3. Archean
4. Hadean

Select the correct answer using the codes below.


A. 1234
B. 4321
C. 3412
D. 2431

Correct Answer : B

Answer Justification :

Justification: The vast expanse of geological time has been separated into eras, periods, and
epochs. The numbers included below refer to the beginnings of the division in which the title
appears. The numbers are in millions of years. The named divisions of time are for the most part
based on fossil evidence and principles for relative dating over the past two hundred years. Only
with the application of radiometric dating have numbers been obtained for the divisions observed
from field observations.

42
Total Marks : 200
Online Prelims TEST - 26 (TEXTBOOK)
( InsightsIAS Mock Test Series for UPSC Preliminary Exam 2020 ) Mark Scored : 0

Q Source:

43
Total Marks : 200
Online Prelims TEST - 26 (TEXTBOOK)
( InsightsIAS Mock Test Series for UPSC Preliminary Exam 2020 ) Mark Scored : 0

https://www.insightsonindia.com/wp-content/uploads/2019/06/InsightsonIndia-May-2019-Current-Af
fairs-final.pdf

55 Union Commerce ministry has found problems with the current method of Services Trade
Restrictiveness Index (STRI) which ranks countries based on their services trade policies. Which of
the following have been some of the concerns raised by the Ministry?
1. The index shows a bias towards developed countries in the way its data was captured.
2. All informal sector trade was completely neglected by the index.
3. Services trade is usually regulated by domestic regulations and not border tariffs.

Select the correct answer using the codes below.


A. 1 only
B. 1 and 3 only
C. 2 and 3 only
D. 1, 2 and 3

Correct Answer : B

Answer Justification :

Justification: This question may be similar to this topic already covered in a previous test;
nonetheless important.

Launched in 2014 and computed by the Organisation for Economic Cooperation and Development
(OECD). The database is based on regulations currently in force.

Uses and significance: It helps to identify which policy measures restrict trade. It provides policy
makers and negotiators with information and measurement tools to improve domestic policy
environment, negotiate international agreements and open up international trade in services. It can
also help governments identify best practice and then focus their domestic reform efforts on priority
sectors and measures.

Statement 1: Union Commerce ministry has found problems with the current method of Services
Trade Restrictiveness Index (STRI) which ranks countries based on their services trade policies,
indicating the outcomes are biased and counter-intuitive.

The 2018 edition covers a total of 45 economies (36 OECD and the rest non-OECD) and 22 sectors.

Issues and concerns raised by India: Design issues that render STRI impractical for use. For
example, the index seems to show the Indian services sector as one of the most restrictive,
particularly in policy areas like foreign entry. This seems surprising as since 1991, the one area that
has seen maximum liberalisation in India is FDI.

Statement 2 : Theoretical and empirical inconsistencies in the OECD methodology: For example,
change in regulatory measures in one policy area can lead to dramatic changes in the STRI in
another policy area which is not very useful for policy purposes. Developed country bias: The data
seems to have been generated by rather arbitrary procedures and reflects a developed country bias.

44
Total Marks : 200
Online Prelims TEST - 26 (TEXTBOOK)
( InsightsIAS Mock Test Series for UPSC Preliminary Exam 2020 ) Mark Scored : 0

Statement 3: The problem in services, IIFT Director, Mr. Pant explains, is that for a long time there
wasn’t any way to know whether a country’s policies were restrictive. “Even if you could ascertain
that, one didn’t know what to do about it since services trade is usually regulated by domestic
regulations and not border tariffs.

“There have been surprisingly few attempts at quantifying the restrictiveness of the services trade,”
Mr. Pant said.

Q Source:
https://www.insightsonindia.com/wp-content/uploads/2019/06/InsightsonIndia-May-2019-Current-Af
fairs-final.pdf

https://www.thehindu.com/business/Industry/india-finds-oecd-index-for-services-trade-faulty/article2
7130497.ece

56 India is a signatory and party to the Chemical Weapons Convention (CWC) of the Organization for the
Prohibition of Chemical Weapons (OPCW). Consider the following about the CWC.
1. The Convention is a multi-lateral disarmament treaty which prohibits the development, production,
stock-piling and use of chemical weapons.
2. India was the First State Party to secure the distinction of chemical weapon free state Party by
destructing all its stockpile of its chemical weapons amongst all State Parties of the Convention.

Select the correct answer using the codes below.


A. 1 only
B. 2 only
C. Both 1 and 2
D. None of the above

Correct Answer : C

Answer Justification :

Justification: India is a signatory and party to the Chemical Weapons Convention (CWC), of the
Organization for the Prohibition of Chemical Weapons (OPCW) with Head Quarters at The Hague,
Netherlands. The Convention is a universal, non-discriminatory, multi-lateral, disarmament treaty
which prohibits the development, production, stock-piling and use of chemical weapons and
monitors its elimination in order to secure chemical weapons free world. India signed the treaty at
Paris on 14th day of January 1993.

India, pursuant to provisions of the Convention enacted the Chemical Weapons Convention Act,
2000. As on date, 193 countries are parties to the Convention. India was the First State Party to
secure the distinction of chemical weapon free state Party by destructing all its stockpile of its
chemical weapons amongst all State Parties of the Convention.

Q Source: https://chemicals.nic.in/chemical-weapons-convention

57 Which of these systems was developed by DRDO for autonomous flying for aerial targets?

45
Total Marks : 200
Online Prelims TEST - 26 (TEXTBOOK)
( InsightsIAS Mock Test Series for UPSC Preliminary Exam 2020 ) Mark Scored : 0

A. ABHYAS
B. YUDHA
C. SCANNER
D. ROG

Correct Answer : A

Answer Justification :

Justification: DRDO recently conducted the flight test of ABHYAS – High-speed Expendable Aerial
Target (HEAT) from a test range in Odisha.

The configuration of ABHYAS is designed on an in-line small gas turbine engine and it uses
indigenously developed MEMS based navigation system. ‘Abhyas’ is designed for autonomous flying
with the help of an autopilot.

A Luneburg lens in the nose cone improves the radar cross-section of the target for weapons
practice. It also has an acoustic miss distance indicator (AMDI) to indicate the missed distance.

It offers a realistic threat scenario for practice of weapon systems. Abhyas has RCS, Visual and IR
augmentation systems required for weapon practice. Abhyas was successfully flight tested on 13
May 2019.

Q Source: In news

58 Mission Shakti is a joint programme of the Defence Research and Development Organisation (DRDO)
and the Indian Space Research Organisation (ISRO). As a part of the programme
1. An anti-satellite (A-SAT) weapon was launched for augmenting space warfare capabilities.
2. An asteroid redirection intelligence service was launched to support NASA’s endeavours for
ensuring the safety of passage for satellites.

Select the correct answer using the codes below.


A. 1 only
B. 2 only
C. Both 1 and 2
D. None of the above

Correct Answer : A

Answer Justification :

Justification: Stating that defence and offensive space technologies are being developed with
various aims of spying, gaining control, deactivating service and destroying, French Envoy in India
Alexandre Ziegler has supported India’s Anti-Satellite (ASAT) missile test as a response to these
growing threats.

46
Total Marks : 200
Online Prelims TEST - 26 (TEXTBOOK)
( InsightsIAS Mock Test Series for UPSC Preliminary Exam 2020 ) Mark Scored : 0

Mission Shakti is a joint programme of the Defence Research and Development Organisation
(DRDO) and the Indian Space Research Organisation (ISRO). As part of the mission, an anti-satellite
(A-SAT) weapon was launched and targeted an Indian satellite which had been decommissioned.

Mission Shakti was carried out from DRDO’s testing range in Odisha’s Balasore. Significance: India
is only the 4th country to acquire such a specialised and modern capability, and Entire effort is
indigenous. Till now, only the US, Russia and China had the capability to hit a live target in space.

Q Source: In news

59 Who among the following authorities has the right to determine the strength of a high court from
time to time?

A. President of India
B. Chief Justice of India
C. Union Minister of Law and Justice
D. Chairman, Bar Council of India

Correct Answer : A

Answer Justification :

Learning: Every high court (whether exclusive or common) consists of a chief justice and such
other judges as the president may from time to time deem necessary to appoint.

Thus, the Constitution does not specify the strength of a high court and leaves it to the discretion of
the president who determines it based on its workload.

This is provided for in the Constitution of India, which deals with the organisation, independence,
jurisdiction, powers, procedures and so on of the high courts.

Q Source: Indian Polity: M Laxmikanth

60 Following the three successful rounds of bidding, the Ministry of Civil Aviation has launched the
4th round of Regional Connectivity Scheme (RCS)- Ude Desh Ka Aam Nagrik (UDAN). Consider the
following about it.
1. It aims to enhance connectivity to remote and regional areas of the country such as focussing on
priority areas like North-eastern region, Hilly States, J&K, Ladakh and the Union Territory Islands.
2. Viability Gap Funding (VGF) under the scheme will be restricted to only inter-state routes or long
routes.
3. Airports that has already been developed by AAI would be given higher priority for award of VGF
under the Scheme.

Select the correct answer using the codes below.


A. 1, 2 and 3
B. 1 and 2 only

47
Total Marks : 200
Online Prelims TEST - 26 (TEXTBOOK)
( InsightsIAS Mock Test Series for UPSC Preliminary Exam 2020 ) Mark Scored : 0

C. 1 and 3 only
D. 2 and 3 only

Correct Answer : C

Answer Justification :

Justification: It aims to further enhance the connectivity to remote and regional areas of the
country. Focus of this round would be the priority areas like NER, Hilly States, J&K, Ladakh and
Islands. Some of the key features of the Scheme are:

Revision of (Viability Gap Funding) VGF cap – The provision of VGF for Category 2 / 3 aircraft
(more than 20 seater) has been enhanced for operation of RCS flights in Priority Area(s) (Union
Territories [UT] of Ladakh and Jammu & Kashmir; the States of Himachal Pradesh, Uttarakhand,
North Eastern State; UT of Lakshadweep and Andaman & Nicobar). The VGF cap applicable for
various stage lengths for operation through category 1 / 1 A aircraft (below 20 seater) has also been
revised to further incentivize the operation of small aircraft under the Scheme.

Promoting short-haul routes – The provision of VGF would be restricted for routes with stage
length up to 600 kms, for operation of Category 2 / 3 aircraft, beyond it no monetary support would
be provided. The table for provision of VGF for various stage lengths would be available for stage
length up to 500 kms.

Well defined Prioritization framework – Airports that has already been developed by AAI would
be given higher priority for award of VGF under the Scheme, followed by airports not part of the
above list but located Priority Area(s) would be given a priority, followed by airports located in
areas other than Priority Area(s).

Flexibility to change the frequency of flight operation - The Selected Airline Operator (SAO)
would be allowed to change the frequency of flight operation, during the tenure of flight operation
of the given route, provided that the total scheduled flight operation submitted as part of the
Technical Proposal, is conformed and adhered to within a period of one year.

Inclusion of helicopter and sea plane operation under NSOP license – The operation of
helicopter and sea plane would be allowed under this round.

Q Source: https://pib.gov.in/newsite/PrintRelease.aspx?relid=195351

https://www.civilaviation.gov.in/rcs

61 Consider the following statements.


1. Most of the rivers of the Indo-Gangetic Plains are of dendritic type.
2. Dendritic pattern develops in a terrain which has extremely varied lithology.

Select the correct answer using the codes below.


A. 1 only
B. 2 only

48
Total Marks : 200
Online Prelims TEST - 26 (TEXTBOOK)
( InsightsIAS Mock Test Series for UPSC Preliminary Exam 2020 ) Mark Scored : 0

C. Both 1 and 2
D. None of the above

Correct Answer : A

Answer Justification :

Justification: Dendritic drainage pattern gives the appearance of branching looking like a tree.

Thus, a dendritic pattern develops in a terrain which has uniform lithology, and where faulting and
jointing are insignificant; e.g., massive crystalline rocks or thick plains consisting of clays.

This is why most of the rivers of the Indo-Gangetic Plains are of dendritic type.

It is a distinctive feature of the regions having horizontally bedded sedimentary rocks or massive
igneous rocks.

For details about other drainage patterns, you can refer the Q Source.

Q Source: AR: 9th NCERT Geography: Chapter on Drainage

62 Consider the following statements.


The National Digital Communications Policy, 2018
1. advocates establishment of a National Digital Grid by creating a National Fibre Authority
2. aims to provide universal broadband connectivity at 50 Mbps to every citizen
3. intends to expand the Internet of Things (IoT) ecosystem
4. advocates facilitating development of Open Access Next Generation Networks

Select the correct answer using the codes below.


A. 1, 3 and 4 only
B. 3 and 4 only
C. 1, 2, 3 and 4
D. 1 and 2 only

Correct Answer : C

Answer Justification :

Justification: Statement 2 and 3: The NDCP-2018 envisions supporting India's transition to a


digitally empowered economy and society by fulfilling the information and communications needs of
citizens and enterprises by establishment of a ubiquitous, resilient and affordable digital
communications infrastructure and services.

The ‘Customer focused’ and ‘application driven’ NDCP-2018 shall lead to new ideas and
innovations, after the launch of advanced technology such as 5G, IOT, M2M, etc. which shall govern
the telecom sector of India.

49
Total Marks : 200
Online Prelims TEST - 26 (TEXTBOOK)
( InsightsIAS Mock Test Series for UPSC Preliminary Exam 2020 ) Mark Scored : 0

Objectives:

The key objectives of the policy are:

1. Broadband for all;

2. Creating four million additional jobs in the Digital Communications sector;

3. Enhancing the contribution of the Digital Communications sector to 8% of India's GDP from ~
6% in 2017;

4. Propelling India to the Top 50 Nations in the ICT Development Index of ITU from 134 in 2017;

5. Enhancing India's contribution to Global Value Chains; and

6. Ensuring Digital Sovereignty.

These objectives are to be achieved by 2022.

Features:

The policy aims to

Provide universal broadband connectivity at 50 Mbps to every citizen;

Provide 1 Gbps connectivity to all Gram Panchayats by 2020 and 10 Gbps by 2022;

Ensure connectivity to all uncovered areas;

Attract investments of USD 100 billion in the Digital Communications Sector;

Train one million manpower for building New Age Skill;

Expand IoT ecosystem to 5 billion connected devices;

Establish a comprehensive data protection regime for digital communications that safeguards
the privacy, autonomy and choice of individuals

50
Total Marks : 200
Online Prelims TEST - 26 (TEXTBOOK)
( InsightsIAS Mock Test Series for UPSC Preliminary Exam 2020 ) Mark Scored : 0

Facilitate India's effective participation in the global digital economy;

Enforce accountability through appropriate institutional mechanisms to assure citizens of safe


and

Secure digital communications infrastructure and services

Statement 1 and 4: Strategy:

The policy advocates:-

1. Establishment of a National Digital Grid by creating a National Fibre Authority;

2. Establishing Common Service Ducts and utility corridors in all new city and highway road
projects;

3. Creating a collaborative institutional mechanism between Centre, States and Local Bodies for
Common Rights of Way, standardization of costs and timelines;

4. Removal of barriers to approvals; and

5. Facilitating development of Open Access Next Generation Networks.

Q Source: https://dot.gov.in/relatedlinks/national-digital-communications-policy-2018

https://pib.gov.in/newsite/PrintRelease.aspx?relid=183711

63 Which one of the following can be identified as a ‘Hard Currency’?

A. Venezuelan Bolivar
B. Japanese Yen
C. Iranian Rial
D. Sri Lankan Rupee

Correct Answer : B

Answer Justification :

Justification: It is the international currency in which the highest faith is shown and is needed by
every economy.

51
Total Marks : 200
Online Prelims TEST - 26 (TEXTBOOK)
( InsightsIAS Mock Test Series for UPSC Preliminary Exam 2020 ) Mark Scored : 0

It is always scarce. Some of the best hard currencies of the world today are the US dollar, the
Euro(€), Japanese Yen (¥) and the UK Sterling Pound (£).

The strongest currency of the world is one which has a high level of liquidity. Basically, it belongs to
the economy with the highest as well as highly diversified exports that are compulsive imports for
other countries.

This can be due to high level technology, defence products, life saving medicines and petroleum
products) will also create high demand for its currency in the world and become the hard currency.

Q Source: Indian Economy: Ramesh Singh

64 Totipotency is the potential of

A. networking between cells based on their mRNA


B. cells to divide rapidly via budding
C. storing large volumes of information within a cell
D. undifferentiated plant tissues to differentiate into functional plants when cultured in vitro

Correct Answer : D

Answer Justification :

Learning: Plant tissue culture is the culture and maintenance of plant cells or organs in sterile,
nutritionally and environmentally supportive conditions (in vitro).

It includes the cultural techniques for regeneration of functional plants from embryonic tissues,
tissue fragments, isolated cells etc.

In commercial settings, tissue culture is often referred to as micro-propagation, which is in fact one
of the techniques in tissue culture.

Micro-propagation refers to the production of whole plants from cell cultures derived from explants
(the initial piece of tissue put into culture) or meristem cells.

You can read this to understand it in detail


http://passel.unl.edu/pages/informationmodule.php?idinformationmodule=957885612&topicorder=
4&maxto=8

Q Source: 12th Biology NCERT

65 The responsibility of maintaining local law and order in the disturbed North-Eastern regions of the
country rests primarily with the

A. Central Government
B. State Governments
C. Tribal Councils established under Sixth Schedule of Constitution

52
Total Marks : 200
Online Prelims TEST - 26 (TEXTBOOK)
( InsightsIAS Mock Test Series for UPSC Preliminary Exam 2020 ) Mark Scored : 0

D. Autonomous District Councils

Correct Answer : B

Answer Justification :

Learning: Law and order is a state subject. However the efforts of State Government are often
supplemented by the Central Government.

Deployment of CAPF, Border security forces like Assam Rifles and ITBP are such examples.

The state government can also request Central assistance in case of major public order problems
such as insurgencies.

Q Source: Basics: Polity

66 The officer Suta in the later Vedic period was responsible for

A. Issuing diplomatic passports for government officials


B. Collection of taxes
C. Charioteers
D. Record keeping of the King

Correct Answer : C

Answer Justification :

Learning: During the early Vedic period the seeds of regular system of administration were sown.

The king had certain officials like, Mahishi (Chief queen), Suta (Charioteers), Bhagadudha
(Chief Revenue Collector), Akshavapa (Chief Accountant), Kaata (Lord of the Imperial
Household), Sangrahita (the Exchequer), Govikruta (Chief of forests) and Palagala.

The Purohit was the Chief Adviser and temporal guru (spiritual teacher) of the king.

The Senani was the military Commander while the Gramina was the village officer. The
importance of these two officials is evident from the fact that the king had to take oath in the
presence of these officials.

Q Source: AR: Past year UPSC papers

67 With reference to the role of the Telecom Regulatory Authority of India (TRAI), consider the following
statements.

53
Total Marks : 200
Online Prelims TEST - 26 (TEXTBOOK)
( InsightsIAS Mock Test Series for UPSC Preliminary Exam 2020 ) Mark Scored : 0

1. TRAI regulates telecom services, including fixation/revision of tariffs for telecom services.
2. One of the objectives of TRAI is to provide a fair and transparent policy environment promoting a
level playing field amongst players and facilitating fair competition.

Select the correct answer using the codes below.


A. 1 only
B. 2 only
C. Both 1 and 2
D. None of the above

Correct Answer : C

Answer Justification :

Justification: The entry of private service providers brought with it the inevitable need for
independent regulation. The Telecom Regulatory Authority of India (TRAI) was, thus, established in
1997 by an Act of Parliament, called the Telecom Regulatory Authority of India Act, 1997, to
regulate telecom services, including fixation/revision of tariffs for telecom services which were
earlier vested in the Central Government.

TRAI's mission is to create and nurture conditions for growth of telecommunications in the country
in a manner and at a pace which will enable India to play a leading role in emerging global
information society.

One of the main objectives of TRAI is to provide a fair and transparent policy environment which
promotes a level playing field and facilitates fair competition.

In pursuance of above objective TRAI has issued from time to time a large number of regulations,
orders and directives to deal with issues coming before it and provided the required direction to the
evolution of Indian telecom market from a Government owned monopoly to a multi operator multi
service open competitive market.

The directions, orders and regulations issued cover a wide range of subjects including tariff,
interconnection and quality of service as well as governance of the Authority.

Learning: The TRAI Act was amended by an ordinance, effective from 24 January 2000,
establishing a Telecommunications Dispute Settlement and Appellate Tribunal (TDSAT) to take over
the adjudicatory and disputes functions from TRAI. TDSAT was set up to adjudicate any dispute
between a licensor and a licensee, between two or more service providers, between a service
provider and a group of consumers, and to hear and dispose of appeals against any direction,
decision or order of TRAI.

Q Source: https://main.trai.gov.in/about-us/history

68 Consider the following about “Principles of Natural Justice”.


1. No one should be made a judge in his own case, or the rule against bias
2. No one should be condemned unheard or without the rule of fair hearing.

54
Total Marks : 200
Online Prelims TEST - 26 (TEXTBOOK)
( InsightsIAS Mock Test Series for UPSC Preliminary Exam 2020 ) Mark Scored : 0

Which of the above is/are correct?


A. 1 only
B. 2 only
C. Both 1 and 2
D. None

Correct Answer : C

Answer Justification :

Justification: Two core points in the concept of principles of natural justice are

No one should be made a judge in his own case, or the rule against bias

Rule that no one should be condemned unheard or the rule of fair hearing

Statement 1: The hearings in all courts are based on these two fundamental principles.

The judge is an impartial observer and allows the prosecution and defence to present their sides
before arriving at a judgment.

Statement 2: The expression “Natural Justice” hasn’t been explicitly used in the constitution.
However, the idea passes through the body of Indian constitution.

Apart from preamble, A14 ensures equality before law and equal protection of law to the
citizen of India striking at the root of arbitrariness.

A21 guarantees right to life and liberty which is the fundamental provision to protect liberty
and ensure life with dignity.

Art 22 guarantees natural justice and provision of fair hearing to the arrested person.

Q Source: Fundamentals of Law

69 The Deccan Plateau is largely occupied with

A. Basalt rocks and red Soil


B. Granite rocks and laterite Soil
C. Igneous rocks and black soil
D. Sedimentary rocks and alluvial Soil

55
Total Marks : 200
Online Prelims TEST - 26 (TEXTBOOK)
( InsightsIAS Mock Test Series for UPSC Preliminary Exam 2020 ) Mark Scored : 0

Correct Answer : C

Answer Justification :

Learning: Since the Deccan plateau is mainly made of igneous rocks, black soil is common.

The black soil is also known as Regur soil which is best soil for cotton cultivation.

It is a mature soil and has high water retaining capacity. So, it swells and will become sticky
when wet and shrink when dried.

Self-ploughing is a characteristic of the black soil as it develops wide cracks when dried.

It is rich in Iron, lime, calcium, potassium, aluminum and magnesium, and deficient in
Nitrogen, Phosphorous and organic matter.

Q Source: 9th Geography NCERT

70 e-Vidhan is a Mission Mode Project (MMP) under the Digital India Programme which aims to bring all
the legislatures of the country together in one platform. The nodal ministry for e-Vidhan is

A. Ministry of Home Affairs


B. Ministry of Parliamentary Affairs
C. Ministry of Law and Justice
D. Ministry of Electronics and Information Technology

Correct Answer : B

Answer Justification :

Justification: Aim of the project: To bring all the legislatures of the country together, in one
platform thereby creating a massive data depository without having the complexity of multiple
applications.

It is a Mission Mode Project (MMP) comes under the Digital India Programme. Ministry of
Parliamentary Affairs (MoPA) is the ‘Nodal Ministry’ for its implementation in all the 31 States/UTs
with Legislatures.

The funding for e-Vidhan is provided by the MoPA and technical support by Ministry of Electronics
and Information Technology (MietY). The funding of NeVA is through Central Sponsored Scheme.
60:40; and 90:10 for North East & hilly States and 100% for UTs.

Q Source: In news

56
Total Marks : 200
Online Prelims TEST - 26 (TEXTBOOK)
( InsightsIAS Mock Test Series for UPSC Preliminary Exam 2020 ) Mark Scored : 0

71 Among the following, which one is NOT an objective of food management in India?

A. Distributing food grains to consumers at affordable prices


B. Procuring food grains from farmers at remunerative prices
C. Maintaining of food buffers for food security and price stability
D. Allocating major portion of the harvest for food processing and retailing

Correct Answer : D

Answer Justification :

Justification: India processes a small fraction, hardly 8-10%, of its harvest, so statement D is
incorrect. It is not an objective of food management in India.

However, it is the government’s policy to process more and more foods items as food processing
sector is a sunrise sector and contributes substantially to employment after sectors like agriculture
and textiles.

Q Source: Department of Food and Public Distribution

72 Consider the following statements.


1. Limestone mining may result in release of the green house gas carbon dioxide.
2. Limestone deposits often occur in association with karst.

Which of the above is/are correct?


A. 1 only
B. 2 only
C. Both 1 and 2
D. None

Correct Answer : C

Answer Justification :

Justification: Statement 1: Limestone releases carbon dioxide upon heating to form calcium oxide,
commonly called quicklime. Mining and processing of lime aids this reaction.

Statement 2: Limestone deposits often occur in association with karst, a topography where
limestone slowly dissolves away underground.

The deposits result in sinkholes, caves and areas of rock fractures that form underground
drainage areas.

When mining occurs in karst, it can cause disruption to natural aquifers, or flows of

57
Total Marks : 200
Online Prelims TEST - 26 (TEXTBOOK)
( InsightsIAS Mock Test Series for UPSC Preliminary Exam 2020 ) Mark Scored : 0

underground water.

Q Source: AR: 10th NCERT Geography

73 Which of the following international organizations works as a cartel?


⌰〰〰

A. World Trade Organization (WTO)


B. North American Free Trade Area (NAFTA)
C. International labour Organization (ILO)
D. Organization of Petroleum Exporting Countries (OPEC)

Correct Answer : D

Answer Justification :

Justification: A cartel is an association of manufacturers or suppliers with the purpose of


maintaining prices at a high level and restricting competition.

NAFTA is a free trade bloc. ILO and WTO are inter-governmental organizations.

OPEC is a notorious cartel that has often spiked oil prices by cutting supplies.

Domestic cartel formation is usually restricted by anti-competition laws, but international cartels
cannot be regulated as such.

Q Source: International Organizations

74 Highest ‘Carbon content’, among the following, will be found in

A. Oceans store more carbon than the atmosphere.


B. Lithosphere has less carbon stored than the oceans.
C. Both (a) and (b)
D. None of the above

Correct Answer : A

Answer Justification :

Learning: Carbon can be stored in and exchanges between particulate and dissolved inorganic and
organic forms and exchanged with the atmosphere as CO2.

This is why the oceans store much more carbon than the atmosphere and the terrestrial biosphere
(plants and animals).

58
Total Marks : 200
Online Prelims TEST - 26 (TEXTBOOK)
( InsightsIAS Mock Test Series for UPSC Preliminary Exam 2020 ) Mark Scored : 0

The carbon, however, requires centuries to penetrate into the deep ocean, because the mixing of
the oceans is a rather slow

However, even more carbon is stored in the lithosphere, i.e. the rocks on the planet, including
limestones (calcium carbonate, CaCO3).

Q Source: http://worldoceanreview.com/en/wor-1/ocean-chemistry/co2-reservoir/

75 Which of the following matches is correct?

A. Bhur denotes an elevated piece of land situated along the banks of the Ganga river
B. Bhangar and Khadar are old and new alluvium soils respectively.
C. Both (a) and (b)
D. None of the above

Correct Answer : C

Answer Justification :

Justification: Bhur denotes an elevated piece of land situated along the banks of the Ganga river
especially in the upper Ganga-Yamuna Doab. This has been formed due to accumulation of wind-
blown sands during the dry hot months of the year.

Khadar and Bhangar are new and old alluvium soils respectively.

Reh or Kallor comprises barren saline efflorescences of drier areas in Uttar Pradesh and Haryana.

Reh areas have spread in recent areas with increase in irrigation.

Q Source: Based on past year UPSC papers

76 Consider the following about Biopharmaceuticals.


1. These are proteins or nucleic acids drugs directly extracted from native biological source.
2. They can be extracted from transgenic organisms.

Select the correct answer using the codes below.


A. 1 only
B. 2 only
C. Both 1 and 2
D. None of the above

Correct Answer : B

Answer Justification :

Justification: Statement 1: They are proteins (including antibodies), nucleic acids (DNA, RNA or
59
Total Marks : 200
Online Prelims TEST - 26 (TEXTBOOK)
( InsightsIAS Mock Test Series for UPSC Preliminary Exam 2020 ) Mark Scored : 0

antisense oligonucleotides) used for therapeutic or in vivo diagnostic purposes, and are produced by
means other than direct extraction from a native (non-engineered) biological source.

Statement 2: The large majority of biopharmaceutical products are pharmaceuticals that are
derived from life forms.

A potentially controversial method of producing biopharmaceuticals involves transgenic organisms,


particularly plants and animals that have been genetically modified to produce drugs.

The government sometime back approved a programme for development of bio pharmaceuticals
through industry-academia collaboration, entailing an investment of Rs 1,500 crore by it over a
period of five years.

The programme will focus on development of specific products such as vaccines, biotherapeutics,
medical devices and diagnostics.

Q Source: Based on past year UPSC papers

77 The Islington Commission, 1917, was concerned with

A. Imperial police service


B. Drought conditions
C. Uniform land codes
D. University education

Correct Answer : A

Answer Justification :

Justification: The commission believed that entry to the Indian Imperial Police Service be thrown
open also to select Indians — those who had received their education in England.

The Commission also recommended that the Police should have a cadre of their own and should not
be headed by outsiders.

In 1924, it was proposed that the proportion of Indian officers in the Imperial Service should be
fixed at 50%.

From 1921, direct appointments to the Indian Imperial Police were also made by an annual
examination in India, open to all races, and over the next 26 years, the service was progressively
Indianised.

Q Source: History of All India Civil Services

78 Consider the following statements.


‘Soft Loan’
1. is granted exclusively to least developed nations

60
Total Marks : 200
Online Prelims TEST - 26 (TEXTBOOK)
( InsightsIAS Mock Test Series for UPSC Preliminary Exam 2020 ) Mark Scored : 0

2. is given on comparatively lenient terms than the loans available in the market

Which of the above is/are correct?


A. 1 only
B. 2 only
C. Both 1 and 2
D. None

Correct Answer : B

Answer Justification :

Learning: It can be given to any nation. The easier conditions in a soft loan might be in the form of
lower interest rates, prolonged repayment duration, etc.

The repayment of these soft loans might also include interest holidays. This process of
extending soft loans is also known as soft financing or concessional funding.

As the loans extended are at much easier terms, these are generally not provided by private
financial institutions. They are primarily provided by government agencies.

For e.g. India recently committed soft loans amounting to USD 340 million to Nepal.

Q Source:
http://timesofindia.indiatimes.com/business/india-business/SIDBI-offering-soft-loan-to-small-medium
-businesses/articleshow/53898336.cms

79 In a Permaculture Garden

A. You are unlikely to see food plants in strict rows as is the case in a commercial farm.
B. The planters focus on closing the nutrient and water loop by using waste, and reducing the
dependence on inputs.
C. Both (a) and (b)
D. None of the above

Correct Answer : C

Answer Justification :

Learning: A permaculture garden is more than an organic garden.

It is a system of agricultural and social design principles centered around simulating or

61
Total Marks : 200
Online Prelims TEST - 26 (TEXTBOOK)
( InsightsIAS Mock Test Series for UPSC Preliminary Exam 2020 ) Mark Scored : 0

directly utilizing the patterns and features observed in natural ecosystems.

In a permaculture garden, you are unlikely to see food plants in rows. It imitates nature.
There is an intensive use of space. Plants are allowed to set seed and are inter-planted for
pest control.

It also has a focus on closing the nutrient and water loop by using waste, and reducing the
dependence on inputs.

Such an Intelligent design uses free, sustainable energies and resources. It is energy-wise and
collaborative to minimise the impact of a site on the surrounding environment.

You can understand more here


http://www.permaculturevisions.com/difference-between-organic-gardening-and-permaculture/

Q Source: Improvisation: http://www.fao.org/agroecology/database/detail/en/c/882864/

80 Which of the following actions would worsen a Balance of Payments (BoP) crisis?

A. Raising import duties


B. Borrowing from abroad
C. Blocking the flow of outward remittances from India
D. None of the above

Correct Answer : B

Answer Justification :

Justification: BoP deficit can be caused from both current account side as well as capital account
side.

Option A: If imports are high, it causes trade deficit and tends to cause a BoP deficit. Restricting
imports by hiking import duty is a short-term way to improve BoP.

Option B: Borrowing abroad would increase capital account deficit and further inflate BoP deficit.

Option C: If you block the flow of outward remittance, less money is flowing out of your country,
which can help tackle the BoP. If inward remittances also increase, it will further help addressing
BoP problems.

Q Source: 12th Macroeconomics NCERT

81 Rhinoceros in India can be found in

62
Total Marks : 200
Online Prelims TEST - 26 (TEXTBOOK)
( InsightsIAS Mock Test Series for UPSC Preliminary Exam 2020 ) Mark Scored : 0

A. Gir and Bharatpur


B. Khaziranga and Dudhwa
C. Nilgiri and Orang
D. Jaldapara and Gir

Correct Answer : B

Answer Justification :

Learning: Kaziranga National Park is famous for Great Indian one Horned Rhino.

In India rhinos are found in Kaziranga, Orang, Pobitara, Jaldapara, Dudhwa.

The great one-horned rhino could once be found from Pakistan all the way through India,
Nepal, Bangladesh, Bhutan and Myanmar.

By the turn of the century, this species had vanished from much of its range, and today only
about 2500 survive in India and Nepal.

Throughout their range, their habitat continues to dwindle fast due to conversion of grassland
habitats into agricultural fields and other human pressures.

Q Source: http://www.wwfindia.org/one_horned_rhino.cfm

82 The Central Asian state that does NOT border China is

A. Kyrgyzstan
B. Uzbekistan
C. Kazakhastan
D. Tajikistan

Correct Answer : B

Answer Justification :

Learning: Both Turkmenistan and Uzbekistan do not border China. In South Asia, states like
Afghanistan, India, Pakistan, Nepal and Sikkim border China.

63
Total Marks : 200
Online Prelims TEST - 26 (TEXTBOOK)
( InsightsIAS Mock Test Series for UPSC Preliminary Exam 2020 ) Mark Scored : 0

Q Source: Map-based questions

83 Identify the correct statements:

A. A writ of Habeas Corpus for the release of a person can be issued by the court when the
arrest or detention has taken place in contravention of the procedure established by law
B. The writ of habeas corpus can be issued against both public authorities as well as private
individuals.
C. Both (a) and (b)
D. None of the above

Correct Answer : C

Answer Justification :

Learning: It is an order issued by the court to a person who has detained another person, to
produce the body of the latter before it.

The court then examines the cause and legality of detention. It would set the detained person
free, if the detention is found to be illegal. So, a is correct.

Thus, this writ is a bulwark of individual liberty against arbitrary detention.

The writ of habeas corpus can be issued against both public authorities as well as private
individuals.

The writ, on the other hand, is not issued where the (a) detention is lawful, (b) the proceeding
is for contempt of a legislature or a court, (c) detention is by a competent court, and (d)
detention is outside the jurisdiction of the court.

64
Total Marks : 200
Online Prelims TEST - 26 (TEXTBOOK)
( InsightsIAS Mock Test Series for UPSC Preliminary Exam 2020 ) Mark Scored : 0

Q Source: Chapter on Fundamental Rights: Indian Polity: M Laxmikanth

84 Consider the following statements.


1. At present no separate permission is required for online sale of essential commodities
2. Essential commodities Act is applicable to such commodities being traded either offline or online.

In the context of the above, which of these is correct?


A. 1 only
B. 2 only
C. Both 1 and 2
D. None of the above

Correct Answer : C

Answer Justification :

Justification: At present no separate permission is required for online sale of essential


commodities including foodgrains and grocery items falling under category of essential
commodities, because online as well as offline trade of essential commodities are treated legally on
the same footing for trade purpose.

So, those traders who have license for offline sale of essential commodities including foodgrains
may also sale them online.

To ensure the availability of essential food items in the general retail market, the States being
enforcement agencies for EC Act, 1955, take action against black marketing, hoarding, profiteering
and speculative trading and unscrupulous traders, whether its online or offline.

Q Source: In news sometimes

85 Omega-3 fatty acids, that are important for normal metabolism, can be found in?
1. Plant based sources
2. Animal derived sources
3. Fish and Krills

Select the correct answer using the codes below.


A. 1 and 3 only
B. 2 only
C. 1 only
D. 1, 2 and 3

Correct Answer : D

Answer Justification :

65
Total Marks : 200
Online Prelims TEST - 26 (TEXTBOOK)
( InsightsIAS Mock Test Series for UPSC Preliminary Exam 2020 ) Mark Scored : 0

Justification: Statement 1: Strawberry or Broccoli contain these fatty acids. It is also found in
plant sources such as nuts and seeds.

Statement 2: Meat and egg also contain these acids in significant quantities.

Statement 3: However, the most widely available dietary source and riches sources are oily fish,
such as salmon and herring.

Q Source: AR: Past year UPSC papers

86 The ideal of ‘Vasudhaiva Kutumbakam’ is to be originally found in

A. Vishnu Upanishad
B. Garuda Purana
C. Mahopnishad
D. Mandukya Upanishad

Correct Answer : C

Answer Justification :

Learning: The original verse is contained in the Mahopanishad. Subsequent shlokas go on to say
that those who have no attachments go on to find the Brahman (the one supreme, universal Spirit
that is the origin and support of the phenomenal universe).

The context of this verse is to describe as one of the attributes of an individual who has attained the
highest level of spiritual progress, and one who is capable of performing his wordly duties without
attachment to material possessions.

Q Source: AR: Past year UPSC papers

87 Among the following, the crop that is least drought-resistant is

A. Sesamum
B. Groundnut
C. Sugarcane
D. Millets

Correct Answer : C

Answer Justification :

Justification: The least drought resistant crop would be the one requiring most rainfall for growth.

Option A: This oilseed requires nearly 40-50 cm of rainfall and can tolerate dry conditions.

66
Total Marks : 200
Online Prelims TEST - 26 (TEXTBOOK)
( InsightsIAS Mock Test Series for UPSC Preliminary Exam 2020 ) Mark Scored : 0

Option B: It requires nearly 50-75 cm rainfall.

Option C: This requires extensive irrigation facilities, or 100-150 cm of rainfall. So, it is only grown
in non-arid regions.

Option D: These are hardy crops capable of survival in arid or semi-arid conditions.

Q Source: Based on past year UPSC papers

88 Among the following, the region that is most vulnerable to earthquakes is

A. The Himalayas
B. Gangetic Plains
C. Sundarban Deltas
D. Western Ghats

Correct Answer : A

Answer Justification :

Learning: The Alpine belt that extends from Java to Sumatra through the Himalayas, the
Mediterranean, and out into the Atlantic accounts for about 17 percent of the world's largest
earthquakes.

The states in and around Himalayas have been rated as under Zone V – Highest vulnerability to
earthquakes in India.

Q Source: 11th NCERT: Indian Physical Geography

89 Consider the following statements.


In Mughal India
1. Abwabs were all temporary and circumstantial taxes and impositions levied by the government over
and above regular taxes.
2. Abwabs were not applicable to Zamindars.

Which of the above is/are correct?


A. 1 only
B. 2 only
C. Both 1 and 2
D. None

Correct Answer : A

Answer Justification :

67
Total Marks : 200
Online Prelims TEST - 26 (TEXTBOOK)
( InsightsIAS Mock Test Series for UPSC Preliminary Exam 2020 ) Mark Scored : 0

Learning: Abwab meant all irregular impositions on raiyats above the established assessment of
land in the pargana.

During nawabi period a variety of abwabs were imposed on zamindars, who conveniently
transferred them on to their raiyats.

Under the Regulations of the permanent settlement (1793) all abwabs were consolidated with
rent and subsequently, further impositions of abwabs were strictly prohibited.

Under the permanent settlement Regulations, zamindars were required to give patta to
raiyats specifying to them the exact amount of rent demanded from them.

It was made a punishable offence to impose abwab under any pretext over and above the rent
recorded in the patta.

Q Source: Based on past year UPSC papers

90 Consider the following statements.


1. Karla Cells are a complex of ancient Indian Buddhist rock-cut cave shrines located in Maharashtra.
2. Dashavatara temple is one of the earliest Hindu stone temples still surviving today.

Which of the above is/are correct?


A. 1 only
B. 2 only
C. Both 1 and 2
D. None

Correct Answer : C

Answer Justification :

Justification: Karla Cells are a complex of ancient Indian Buddhist rock-cut cave shrines located in
Maharashtra. The caves house a Buddhist monastery dating back to the 2nd century BC.

The Dashavatara temple is one of the earliest Hindu stone temples still surviving today. Vishnu
Temple shows the ornate beauty seen in Gupta style architecture.

The Kandariya Mahadeva Temple, one of the best examples of temples preserved from the medieval
period in India, is the largest of the western group of temples in the Khajuraho complex which was
built by the Chandela rulers.

Q Source: Based on past year UPSC papers

68
Total Marks : 200
Online Prelims TEST - 26 (TEXTBOOK)
( InsightsIAS Mock Test Series for UPSC Preliminary Exam 2020 ) Mark Scored : 0

91 Correctly match the following colour codes used by the


Survey of India for classification of land uses in India:
1. White Patches: A. Contour lines
2. Red: B. Settlements, huts and buildings
3. Brown C. Cultivated areas
4. Yellow D. Uncultivated land

What is the correct match for the above?


A. 1D, 2B, 3A, 4C
B. 1A, 2C, 3D, 4B
C. 1B, 2A, 3D, 4A
D. 1A, 2B, 3C, 4D

Correct Answer : A

Answer Justification :

Justification: Black: all writings on the map except grid numbers (names, abbreviations such as
DB, RS, P0), river banks, broken ground, dry streams, surveyed trees, heights and their numbering,
railway lines, telephone and telegraph lines, lines of latitude and longitude.

Blue: water features or water bodies (lakes, rivers, wells, ponds, ctc)

Green: wooded and forested areas shown with green wash; orchards, scattered trees and scrubs
shown in green colour.

Note: Surveyed trees are shown in black which serve as landmarks for further survey related work
and are not allowed to be cut.

Yellow: cultivated areas shown with a yellow wash

White patches: uncultivated land

Brown: contour lines, their numbering, form lines and sand features such as hills and dunes

Red: grid lines and their numbering; wads, cart and foot tracks, settlements, huts and buildings.

Q Source: Based on past year UPSC papers

http://tcpomud.gov.in/Divisions/MUTP/Land/Landuse_Classification_Report.pdf

92 Article 310 and Article 311 of the constitution, sometimes seen in news editorials, are concerned with

69
Total Marks : 200
Online Prelims TEST - 26 (TEXTBOOK)
( InsightsIAS Mock Test Series for UPSC Preliminary Exam 2020 ) Mark Scored : 0

A. Election to Rajya Sabha


B. President’s power of pardoning
C. Instruction in Mother tongue
D. Service conditions of Civil Services

Correct Answer : D

Answer Justification :

Articles: A310 essentially conveys that civil servants of the Union hold office during the pleasure of
the President, and those at the state at the pleasure of the Governor.

A311 conveys that they cannot be removed by an authority subordinate to the one that appointed
them, and that due inquiry must be conducted where they are allowed to hear charges against them
and defend themselves appropriately.

Justification: It is believed by many, including the 2nd ARC that these articles provide undue
protection to dishonest and corrupt civil servants and shield them from action. This is because the
protection is excessive, and hearings go through many bureaucratic layers, which ultimately
corrodes the fabric of administration and its efficiency.

There are many arguments against this as well, but delving into there is not a subject of concern
here.

Q Source: Based on past year UPSC papers

93 Among the following cities, Tropic of Cancer is farthest from

A. Bhopal
B. Ranchi
C. Kolkata
D. Agra

Correct Answer : D

Answer Justification :

Learning: Agra is a bit on the northern side: 27.1767° N, 78.0081° E. You can also guess this easily
since Delhi is pretty close to Agra and Delhi itself is not close to Tropic of Cancer.

Some cities that are closest to tropic of cancer are:

Gandhinagar (Gujarat) - 23.10 N

Bhopal (M.P) - 23.16 N

70
Total Marks : 200
Online Prelims TEST - 26 (TEXTBOOK)
( InsightsIAS Mock Test Series for UPSC Preliminary Exam 2020 ) Mark Scored : 0

Ranchi (Jharkhand) - 23.11 N

Kolkata (W.B) - 22.34 N

Agartala (Tripura) - 23.51 N

Aizwal (Mizoram) - 23.36 N

Q Source: Map based questions

94 It is also known as the ‘coral group of islands’ of India:

A. Minicoy
B. Lakshadweep
C. Andaman
D. Nicobar

Correct Answer : B

Answer Justification :

Learning: During the reorganization of Indian states, it was separated from Madras and organized
into a separate union territory for administrative purposes.

There are no conclusive theories about the formation of these coral atolls. The most accepted theory
is given by the English Evolutionist Sir Charles Darwin.

He concluded in 1842 that the subsidence of a volcanic island resulted in the formation of a fringing
reef and the continual subsidence allowed this to grow upwards.

When the volcanic island became completely submerged the atoll was formed encircling the lagoon
where, with the action of the wind, waves, reef to currents and temperature, the coral islands were
formed.

Q Source: 6th NCERT Geography

95 Consider the following statements.


1. Non-Banking Financial Companies (NBFCs) cannot accept demand deposits.
2. Deposit insurance facility of Deposit Insurance and Credit Guarantee Corporation is not available to
depositors of NBFCs.

Which of the above is/are correct?


A. 1 only

71
Total Marks : 200
Online Prelims TEST - 26 (TEXTBOOK)
( InsightsIAS Mock Test Series for UPSC Preliminary Exam 2020 ) Mark Scored : 0

B. 2 only
C. Both 1 and 2
D. None

Correct Answer : C

Answer Justification :

Justification: NBFCs lend and make investments and hence their activities are akin to that of
banks; however there are a few differences as given below:

NBFC cannot accept demand deposits; and moreover, as they do not form part of the payment
and settlement system, they cannot issue cheques drawn on itself.

Deposit insurance facility of Deposit Insurance and Credit Guarantee Corporation is not
available to depositors of NBFCs, unlike in case of banks.

A Non-Banking Financial Company (NBFC) is a company registered under the Companies Act, 1956
engaged in the business of loans and advances, acquisition of
shares/stocks/bonds/debentures/securities issued by Government or local authority or other
marketable securities.

Q Source: https://www.rbi.org.in/Scripts/FAQView.aspx?Id=92

96 Consider the following statements about National Commission for Women (NCW).
1. It has all the powers of a civil court.
2. It can attend to cases of harassment of women and order appropriate punishments.
3. It can review the Constitutional and Legal safeguards for women.

Which of the above is/are correct?


A. 2 and 3 only
B. 1 only
C. 1 and 3 only
D. None of the above

Correct Answer : C

Answer Justification :

Justification: The National Commission for Women was set up as statutory body in January 1992
under the National Commission for Women Act, 1990 ( Act No. 20 of 1990 of Govt.of India ) to :

review the Constitutional and Legal safeguards for women ;

72
Total Marks : 200
Online Prelims TEST - 26 (TEXTBOOK)
( InsightsIAS Mock Test Series for UPSC Preliminary Exam 2020 ) Mark Scored : 0

recommend remedial legislative measures ;

facilitate redressal of grievances and

advise the Government on all policy matters affecting women.

It is an advisory body. It can attend to grievances but only recommend action, and not order
punishments.

Q Source: http://wcd.nic.in/

97 Which one of the following statements about the ‘polar jet streams’ is true?

A. These winds are strongest in tropopause.


B. Polar fronts and polar jet streams are the same phenomena.
C. Both (a) and (b)
D. None of the above

Correct Answer : A

Answer Justification :

Justification: Option A: It is a belt of powerful upper-level winds that sits atop the polar front. So,
C is incorrect.

The winds are strongest in the tropopause, which is the upper boundary of the troposphere, and
move in a generally westerly direction in midlatitudes.

Option B: They are different phenomena.

Jet streams significantly affect surface airfiows. When they accelerate, divergence of air occurs at
the altitude of the jet stream; this promotes convergence near the surface and induction of cyclonic
motion. Jet streams supply energy to surface storms and direct their path.

Jet streams also cause a convergence of air aloft and subsidence near the surface, resulting in
intensification of high-pressure systems. As such, jet streams are often described as weather
makers.

Q Source: http://climate.ncsu.edu/edu/k12/.atmosphere_circulation

98 Consider the following statements.


1. Albedo is the fraction of solar energy reflected from the Earth back into space.
2. Ice has lesser albedo than water.

73
Total Marks : 200
Online Prelims TEST - 26 (TEXTBOOK)
( InsightsIAS Mock Test Series for UPSC Preliminary Exam 2020 ) Mark Scored : 0

Which of the above is/are correct?


A. 1 only
B. 2 only
C. Both 1 and 2
D. None

Correct Answer : A

Answer Justification :

Justification: Albedo is the fraction of solar energy reflected from the Earth back into space.

It is a measure of the reflectivity of the earth's surface. Ice, especially with snow on top of it, has a
high albedo: most sunlight hitting the surface bounces back towards space.

Water is much more absorbent and less reflective. So, if there is a lot of water, more solar radiation
is absorbed by the ocean than when ice dominates.

Q Source: 11th NCERT: Fundamentals of Physical Geography

99 During the Cold war period, the term ‘Third World’ referred to

A. Middle Eastern region


B. Poor or developing countries
C. Countries that were not aligned with either USA or Soviet Union
D. Colonies of Europe

Correct Answer : C

Answer Justification :

Learning: Third World countries were labelled during the Cold War to reference those nations that
were not aligned with either the United States or the Soviet Union.

Later, once cold war ended, it came to include many countries with colonial pasts in Africa, Latin
America, Oceania and Asia which were poor and underdeveloped. It became a stereotype to refer to
poor countries as "third world countries".

The term has been subject to evolutionary interpretations. Recently the "Third World" term is also
often taken to include newly industrialized countries like Brazil, India and China now more
commonly referred to as part of BRIC.

Q Source: 12th Contemporary World Politics NCERT

100 Cabinet Committee on Appointments includes

74
Total Marks : 200
Online Prelims TEST - 26 (TEXTBOOK)
( InsightsIAS Mock Test Series for UPSC Preliminary Exam 2020 ) Mark Scored : 0

1. Prime Minister
2. Union Minister of Home affairs
3. Union Minister of Finance
4. Union Minister of Defence

Select the correct answer using the codes below.


A. 1, 2 and 3 only
B. 2 and 4 only
C. 1 and 2 only
D. 1, 3 and 4 only

Correct Answer : C

Answer Justification :

Learning: The Appointments Committee of the Cabinet (ACC) decides appointments to several top
posts under the Government of India.

The committee is composed only of the Prime Minister of India (who is the Chairman), and the
Minister of Home Affairs.

The Establishment Officer’s Division (EO Division) processes all proposals for senior
appointments in the Government of India that require approval of the Appointments
Committee of the Cabinet.

The Establishment Officer (EO) is the ex-officio Member Secretary of the Civil Services Board
that is chaired by the Cabinet Secretary.

Q Source: https://pib.gov.in/Pressreleaseshare.aspx?PRID=1573622

75

You might also like